Sie sind auf Seite 1von 53

Behavioural

-----------
There are only 4 situations where disclosure of patient information without consent
of the patient is allowable:
1. Suspected child or elder abuse (laws for spousal abuse vary by state)
2. Gunshot or stabbing injuries
3. Diagnosing a reportable communicable disease
4. When patients threaten to physically harm themselves or others and have
reasonable ability to carry out the threat in the near future

When an ill patient requests your prayers in an acute setting, it is appropriate to


offer your personal support without interjecting your personal beliefs into the
interaction The overriding goal in these situations is to “do no harm." This can be
achieved by not disagreeing with the patient, not entering into a religious debate
with them, and not displacing responsibility for the care of the patient onto
others.

Fever and change in the color of the sputum are typically observed in patients
sufferina from viral airwav infections.

(Choices A and B) if the patient were incapacitated or incompetent, then decision


making would fall on the next of kin The spouse is the immediate next of kin, but
when there is no spouse or the spouse is unable to make decisions, the next of kin
is an adult child
(Choice C) Ajudge intervenes in medical care decisions when there is no next of kin
available who is competent to make medical decisions for an incapacitated or
incompetent patient In cases such as these the court will appoint a guardian to act
on the patient's behalf.
(Choice E) The hospital ethics committee serves to make recommendations in
situations where a decision about care is not straightforward, but they can not
legally act to make decisions for the patient as they have not been designated
guardians by the court.
Educational Objective:
A patient who is competent and not incapacitated has the right to refuse treatment
by a physician at any time, even if that treatment is life-saving A competent
patient understands their situation as well as the possible consequences of
decisions made in that situation.

Causes of erectile dysfunction


• Psychogenic stressors
• Performance anxiety or depression
• Medications (SSRIs, sympathetic blockers)
• Vascular or neurological impairment
• Genitourinary trauma (eg, prostatectomy)

There is a temporal association between this man's troubled relationship with his
wife and his subsequent erectile dysfunction (ED). Psychogenic causes account for
approximately 10% of ED cases and include performance anxiety, sexual partner
dissatisfaction, marital problems, or other emotional issues. An important clue in
psychogenic impotence is rapid onset. Men who had no sexual difficulty until “one
night when they could not have an erection," followed by persistent ED, almost
always have psychogenic impotence. In contrast, men suffering from organic causes
of impotence complain that failure of sexual function first occurred intermittently
and later became more persistent.
The presence or absence of spontaneous erections is another important diagnostic
clue. The majority of men experience spontaneous erections during REM sleep and
will awaken with an erection, demonstrating the integrity of neurologic reflexes
and corpus cavernosa blood flow. Complete loss of nocturnal erections occurs in men
with neurologic or vascular disease but is not seen with psychogenic impotence
(Choice E)
(Choice A) Sexual desire does not decrease with age. As men age, they typically
have a longer refractory period and take longer to achieve an erection.
(Choice B) The most common medications causing impotence are selective serotonin
reuptake inhibitors and sympathetic blockers (clonidine, methyldopa, beta-
blockers). ACE inhibitors and statins have not been shown to cause impotence
(Choice C) Vasectomy does not cause impotence and would not result in impotence 16
years later. In contrast, recent pelvic trauma, prostate surgery, or priapism can
lead to ED.

While this patient is undergoing emergency treatment for acute myocardial


infarction, a woman who is presumably his wife arrives at the hospital and asks
about his medical condition. The Health Insurance Portability and Accountability
Act (HIPAA) is designed to protect patient privacy, and in many situations, the
patient's explicit consent is required in order to share medical information.
However, a health care provider is allowed to share information without explicit
permission in the following circumstances:
• The patient is present and does not object to sharing the information when
given reasonable opportunity. For instance, when family is visiting, the physician
can ask if it is a good time to discuss medical concerns and proceed to do so if
the patient does not object
• The patient is not present, and the provider determines, based on
professional judgment, that sharing information is in the patient's best interest

Under the Health Insurance Portability and Accountability Act, physicians may
disclose patient information to friends and family members when the patient gives
explicit permission or does not object when given a reasonable opportunity. In an
emergency situation, health care providers can also share medical information when
it is in the patient's best interest based on their professional judgment.

This patient's presenting attitude is hostile and threatening; this is a "difficult


patient experience." On the USMLE, the correct answer in difficult patient
situations will almost universally be to seek further information from the patient
using open-ended questions. This patient may have been rejected or mistreated by
practitioners in the past, and this may be why he is presenting to you as a new
patient; at this point in the interaction you know very little about him The most
appropriate next step in this interaction would be to pacify the patient, explain
to him what your ethical and legal obligations are, and inquire about his condition
By explaining yourself to him you allow him to understand why you will need to ask
questions about his condition, and he will likely cooperate with your exam if he
understands that you are taking him seriously.

The primary physician has instituted proper cholesterol and antiplatelet therapy
and has referred the patient for more specialized care of his hypertension. For
unclear reasons, the primary physician placed the patient on a calcium channel
blocker for his hypertension despite the evidence that beta-blockers and
angiotensin-converting enzyme (ACE) inhibitors help improve outcomes after
myocardial infarction. Calcium channel blockers (eg, verapamil) are potent and
effective anti-hypertensives with anti-anginal properties, but they do not provide
an additional mortality benefit for patients with myocardial infarction.

Pain is subjective, and physicians must use clinical judgment to balance effective
pain management with prevention of drug misuse and abuse. Prescription opioid abuse
has become a significant clinical problem recent years. Signs of misuse include not
taking medication as prescribed, early refill requests, accessing drugs from
multiple doctors or illicit sources, requesting higher doses, and refusing
alternate pain management strategies.
in his condition. He is also showing signs of a possible opioid use disorder, le,
obtaining medications from multiple physicians and developing tolerance (higher
doses needed to provide the same effect). The most appropriate action, given the
circumstances, would be to express concern regarding the patient's escalating use
of opioid medication and explore possible reasons for it An empathic, nonjudgmental
attitude that validates the patient's concern about pain control is essential to
developing a collaborative treatment approach.

Features of anorexia nervosa


Epidemiology • Increased in whites, industrialized countries, athletes,
models • Median onset at age 18; 10:1 female
DSM-5 diagnostic criteria • Weight less than minimally normal, BMI <18.5
kg/m2 • Intense fear of becoming fat or persistent avoidance of weight gain •
Distorted body image, excessive influence of weight on self worth, or
persistent denial of complications related to low body weight • Two subtypes: •
Restricting • Binge eating/purging
Medical complications • Osteoporosis • Amenorrhea • Lanugo or hair
loss • Enlarged parotid glands (if binge/purge type) • Hypotension,
hypothermia, bradycardia • Cardiac atrophy, cardiomyopathy, arrhythmias
©USMLEWortd, LLC

This patient's reduced heart size, compression fracture (suggestive of


osteoporosis), amenorrhea, lanugo (fine downy body hair), and vital sign
derangements (hypotension, hypothermia, bradycardia) are consistent with the
medical complications of anorexia nervosa. Patients with anorexia nervosa restrict
their energy intake to maintain a body weight that is below a minimal level for
age, sex, and developmental trajectory. As a result, they develop an emaciated body
type and signs and symptoms secondary to starvation Patients will often exhibit the
female athlete triad - an eating disorder, amenorrhea, and osteoporosis. Although
amenorrhea is a frequent manifestation of anorexia nervosa, not all patients
exhibit this finding and it is no longer required for diagnosis in DSM-5.
Anorexia nervosa can be divided into 2 subtypes: restricting and binge
eating/purging. In the binge eating/purging subtype, the anorectic patient engages
in purging behavior (self-induced vomiting, misuse of laxatives, diuretics).
Patients who induce vomiting may have puffy cheeks from parotid gland hypertrophy,
dental caries, halitosis, and scars or calluses on the hand from contact with the
teeth (Russell's sign).
(Choice B) Bulimia nervosa is characterized by recurrent episodes of binge
eating/purging and preoccupation with body image. Although it is similar to the
binge eating/purging subtype of anorexia nervosa, the key difference is that
patients with bulimia nervosa maintain their body weight at or above the minimally
normal level. As a result, they do not experience the signs of malnutrition and
starvation that are present in this patient.

Advance care planning ideally should begin in the outpatient setting as a


conversation between the patient and the primary care provider. However, when a
patient is hospitalized, it is essential to inquire about advance directives in the
event that the patient becomes unable to make decisions. As part of the admission
process, patients should be asked whether they have advance directives and informed
about options for creating them if they do not. Some physicians may be reluctant to
discuss these issues, but studies have shown that most patients prefer to have end-
of-life discussions with their caregivers.
Advance directives consist of 2 main components: a living will and a health care
proxy. A living will specifies the patient's end-of-life wishes and often includes
specific directives regarding intubation, cardiopulmonary resuscitation, enteral
feeding, and other life-prolonging interventions. A health care proxy document
allows the patient to designate a specific individual to make health care decisions
should the patient become incapacitated. The proxy decision maker must always make
these decisions in accordance with the patient's wishes, as outlined in the living
will.
This patient presents with classic findings of infectious mononucleosis, which is
caused by the Epstein-Barr virus. Patients present with fever, sore throat,
fatigue, lymphadenopathy, tonsillar exudates, and sometimes hepatosplenomegaly.
Diagnosis is confirmed with a positive monospot test and treatment is mainly
supportive. Patients are advised to refrain from athletic activity to avoid splenic
rupture, a rare but serious and potentially life-threatening complication that can
occur with no or very minor trauma. By playing tackle football, this patient has a
high risk of suffering abdominal trauma with possible splenic rupture. Current
guidelines recommend that athletes wait at least one month before returning to
contact sports.

The overriding duty of all physicians, including team physicians, is to protect the
health and safety of the patient. Preferences expressed by the athlete, team,
coaches, or spectators should not influence the physician's decision on critical
health issues: the physician's judgment should be guided only by medical
considerations. Student athletes typically sign an authorization form permitting
the physician to share health-related information with coaches under the Family
Educational Rights and Privacy Act. Because the team physician is employed by the
university (and not acting as a personal physician), the collected health
information is considered part of the student's university record and possesses an
exception under the Health Insurance Portability and Accountability Act (HIPAA). As
a result, the team physician may communicate with coaches in order to protect a
player from further injury based on best medical judgment.

BIOCHEM
___________________________________________________________________
Niemann-Pick disease Type A is an autosomal recessive disorder most common in
individuals of Ashkenazi Jewish descent. Affected infants present in the first year
of life with hepatosplenomegaly and progressive hypotonia and mental retardation
following a period of normal early development. The cause is deficiency of the
sphingomyelinase enzyme, which in normal individuals is responsible for cleaving
sphingomyelin into phosphorylcholine and ceramide. In infants with Niemann-Pick
disease, sphingomyelin accumulates within phagocytes, producing characteristic
"foamy histiocytes." These foamy-appearing, sphingomyelin-laden histiocytes
accumulate in the liver and spleen causing massive hepatosplenomegaly. Progressive
sphingomyelin accumulation in the central nervous system is responsible for the
neurologic degeneration that occurs. Sphingomyelin deposition in the retina causes
blindness as well. A cherry-red macular spot, similar to that seen in Tay-Sachs
disease, is also often found. Death usually occurs before age three.
(Choice A) In patients with the autosomal recessive condition metachromatic
leukodystrophy, a deficiency of the enzyme arylsulfatase A causes sulfatides to
accumulate within tissues.
(Choice B) In the autosomal recessive condition Tay-Sachs disease, a deficiency of
the enzyme (3-hexosaminidase A causes G,^ ganglioside to accumulate within neurons.
(Choice C) In Fabry disease, an X-linked recessive condition, deficiency of the
enzyme a-galactosidase A causes ceramide trihexoside to accumulate in tissues.
(Choice D) p-Glucosidase is a plant enzyme used for the breakdown of starch.
(Choice E) Neuraminidase (sialidase) deficiency is the cause of human sialidosis.
Neuraminidase is also a surface enzyme found on the influenza virus.
(Choice G) Ceramidase deficiency causes Farber disease, an autosomal recessive
condition characterized by ceramide accumulation within neurons and within
granulomas in the skin.
Educational Objective
In Niemann-Pick disease, deficiency of sphingomyelinase causes abnormal
accumulations of the ceramide phospholipid sphingomyelin and neurologic
deterioration within the first year of life.

This patient's symptoms of abdominal pain, neuropsychiatric manifestations, and


color change in her urine upon standing are the classic presenting features of
acute intermittent porphyria (AIP). In order to understand the pathophysiology of
acute hepatic porphyria, it is important to understand the regulation of heme
synthesis. Eighty five percent of heme is produced by bone marrow erythroid cells
for hemoglobin and the remaining 15% is synthesized in the liver, primarily for the
cytochrome P450 detoxification enzyme system. ALA synthase is an enzyme in the heme
biosynthesis pathway that functions to condense succinyl CoA and glycine to form
delta-aminolevulinic acid, the rate-limiting step in the heme synthesis pathway,
which also requires the coenzyme pyridoxal phosphate. Administration of drugs such
as phenobarbital, griseofulvin, phenytoin, and alcohol decrease the hepatic
concentration of heme by inducing the cytochrome P450 enzyme system.
Porphobilinogen deaminase (hydroxymethylbilane synthase) in the heme biosynthesis
pathway converts porphobilinogen (PBG) to hydroxymethylbilane (HMB). Acute
Intermittent Porphyria (AIP) results from a deficiency of hydroxymethylbilane
synthase, resulting in the accumulation of ALA and PBG, which are toxic to human
tissues.
It is important to remember that enzyme deficiencies in the early steps of
porphyrin synthesis cause neurologic abnormalities without photosensitivity, while
derangements in the latter steps of heme synthesis (after condensation of
porphobilinogen) cause photosensitivity. Exposure of urine to light in patients
excreting excess quantities of porphyrinogen results in light induced formation of
porphyrins and a darkening of the color of the urine.
Treatment of acute attacks of porphyria include control of pain with narcotic
analgesics and medications that decrease the activity of ALA synthetase. Two agents
that are particularly useful in decreasing ALA synthetase activity and abrogating
acute episodes of acute intermittent porphyria are glucose and intravenous heme
preparations. Glucose infusion has profound effects on the intermediary metabolism,
but the other options listed in the question are not responsible for decreasing
symptoms.
Educational Objective:
Glucose loading decreases porphyrin synthesis by repressing ALA synthase activity,
thus alleviating the abdominal pain and neuropsychiatric manifestations of Acute
Intermittent Porphyria (AIP).

Catabolism of isoleucine, valine, threonine, methionine, cholesterol, and odd-chain


fatty acids leads to the formation of propionic acid, which is then converted to
methylmalonic acid by biotin-dependent carboxylation Isomerization of methylmalonyl
CoA forms succinyl CoA, which then enters the TCA cycle. A congenital deficiency of
propionyl CoA carboxylase, the enzyme responsible for the conversion of propionyl
CoA to methylmalonyl CoA, leads to the development of propionic acidemia, as
propionyl CoA accumulates.
Propionic acidemia is clinically characterized by poor feeding, vomiting,
hypotonia, lethargy, dehydration, and an anion gap acidosis. Propionic acid is the
intermediate in the catabolism of branched chain amino acids, such as valine, and
is not produced during the catabolism of the other amino acids listed.
(Choice A) Phenylalanine is converted to tyrosine by the enzyme phenylalanine
hydroxylase, which is defective in phenylketonuria (PKU).
(Choice C) Asparagine is a nonessential amino acid that is catabolized initially to
aspartate by the enzyme asparaginase. In rapidly dividing leukemic cells, the
synthesis of asparagine is impaired, so these cells survive by collecting
asparagine from circulating plasma. L-asparaginase works as an antineoplastic agent
by lowering circulating asparagine levels.
(Choice D) Histidine is an essential amino acid in children. Histidine is
deaminated to urocanic acid, which is then converted to N-forminino-glutamate
(FIGIu). The forminino group of FIGIu is donated to tetrahydrofolate to form
glutamate and forminino-tetrahydrofolate. Oxidative decarboxylation of histidine
forms histamine, which is released by mast cells in Type I Hypersensitivity
reactions.
(Choice E) Proline is a nonessential amino acid that is oxidized to glutamate,
which in turn is transaminated to alpha-ketoglutarate
(Choice F) Lysine is an essential amino acid that is strictly ketogenic. The
metabolism of lysine is unique compared with the other amino acids, as it is not
transaminated as an initial step
Educational Objective:
Propionyl CoA is derived from amino acids (Val, lie, Met, and Thr), odd-numbered
fatty acids, and cholesterol side chains. Congenital deficiency of propionyl CoA
carboxylase, the enzyme responsible for the conversion of propionyl CoA to
methylmalonyl CoA, leads to the development of propionic acidemia.

Ammonia is generated from the metabolism of alpha amino acids and is normally
converted to urea by the urea cycle. The urea cycle involves five enzymatic steps:
two in the mitochondrial matrix and three in the cytosol. One more enzyme that
indirectly participates in the urea cycle is N-acetylglutamate synthetase (NAGS).
The first step of urea cycle combines C02, ammonia, and ATP to form carbamoyl
phosphate in a reaction catalyzed by the enzyme carbamoyl phosphate synthetase I,
the rate-limiting step in the urea cycle. Carbamoyl phosphate synthetase I (CPS)
requires the presence of N-acetylglutamate (NAG), a molecule formed by NAGS, as
this molecule acts as an allosteric activator of CPS. None of the other steps in
the urea cycle require NAG as an activator.
The symptoms seen in the patient in this question stem are the result of toxic
effects of ammonia accumulation within the child’s blood and tissues. The first few
feedings provide a protein load to the infant that results in amino acids being
available for metabolism, but the defect in the urea cycle prevents the disposal of
toxic ammonia from the child's body, leading to lethargy, vomiting, and seizures.
Educational Objective:
N-acetylglutamate is an essential activator of carbamoyl phosphate synthase I and
is formed by the enzyme N-acetylglutamate synthetase from the precursors acetyl-CoA
and glutamate.

Pyridoxine (vitamin Bg) is an essential cofactor for the transamination and


decarboxylation of amino acids, for
gluconeogenesis, and for other essential biochemical processes Transamination
reactions typically occur between an amino acid and an a-keto acid. The amino group
is transferred to the ci-keto acid from the amino acid, and the a-keto acid thereby
becomes an amino acid.
Thus, in this question, oxaloacetate (a-keto acid) reacts with glutamate (amino
acid) to form aspartate (the resulting amino acid) and a-ketoglutarate (the
resulting a-keto acid). Transaminases (aminotransferases) are the enzymes that
catalyze transamination reactions, and pyridoxal phosphate (vitamin Bg) serves as
an
essential cofactor for the transaminase.
(Choice A) Thiamine (vitamin B.,) serves as a coenzyme for a number of important
enzymes including transketolase. a-ketoglutarate dehydrogenase, and pyruvate
dehydrogenase.
(Choice B) Riboflavin (vitamin B2) is used in dehydrogenase reactions involving FMN
and FAD.
(Choice C) Many dehydrogenases use NAD and NADP, which are formed from niacin.
Niacin (vitamin B3. or
nicotinic acid) deficiency is known as pellagra and is classically associated with
the four Ds: dermatitis, dementia, diarrhea, and, if untreated, death.
(Choice E) Biotin is a water-soluble vitamin that is a cofactor for all four
carboxylase enzymes: pyruvate carboxylase, acetyl CoA carboxylase, propionyl CoA
carboxylase, and 3-methylcrotonyl-CoA-carboxylase.
(Choice F) Folic acid is an essential cofactor in nucleic acid synthesis, and a
deficiency of either folate or vitamin B12 results in megaloblastic anemia.
Educational Objective:
Transamination reactions typically occur between an amino acid and an a-keto acid.
The amino group from the amino acid is transferred to the a-keto acid, and the a-
keto acid in turn becomes an amino acid. Pyridoxal phosphate (vitamin Bg) serves as
a cofactor in amino acid transamination and in decarboxylation reactions.

Individuals who consume more than 10 times the Daily Value (Recommended Dietary
Allowance) of vitamin A are prone to developing toxicity and may suffer hepatic
injury so severe as to cause cirrhosis.
Vitamin A toxicity has been subdivided into three syndromes: acute, chronic, and
teratogenic. The signs and symptoms of acute toxicity occur after the ingestion of
a single high dose of vitamin A and include nausea, vomiting, vertigo, and blurred
vision. The signs and symptoms of chronic toxicity occur after the long-term
ingestion of high doses of vitamin A, and include alopecia, dry skin,
hyperlipidemia, hepatotoxicity, hepatosplenomegaly, and visual difficulties.
Papilledema, when present, is suggestive of cerebral edema in the setting of benign
intracranial hypertension (pseudotumor cerebri). Teratogenic effects of excessive
vitamin A ingestion include microcephaly, cardiac anomalies, and fetal death
(especially in the first trimester of pregnancy).
(Choice A) Thiamine deficiency is associated with infantile and adult beriberi, as
well as Wernicke-Korsakoff syndrome in alcoholics.
(Choice B) Niacin deficiency is characterized by the 3 D's of pellagra: (dementia,
dermatitis, and diarrhea).
(Choice C) Vitamin B12 (cobalamin) deficiency is frequently associated with
pernicious anemia The classic presentation of pernicious anemia is an older,
mentally slow woman of northern European descent who is "lemon colored" (anemic and
icteric), has a smooth, shiny tongue indicative of atrophic glossitis, and
demonstrates a shuffling broad-based gait.
(Choice D) Vitamin B2 (riboflavin) deficiency is characterized by cheilosis,
stomatitis, glossitis, dermatitis, corneal vascularization, and ariboflavinosis.
(Choice E) Large doses of vitamin C can give false negative stool guaiac results
and are associated with diarrhea and abdominal bloating. Some studies suggest an
association between high doses of vitamin C and calcium oxalate nephrolithiasis,
though this remains controversial
(Choice F) Large doses of vitamin E have been associated with higher mortality
rates due to hemorrhagic stroke in adults and higher rates of necrotizing
enterocolitis in infants.
Educational Objective:
Vitamin A overuse can result in intracranial hypertension, skin changes and
hepatosplenomegaly.

The child described in this question stem gives a history that is consistent with
lead poisoning. 5* Aminolevulinate dehydrase and ferrochelatase are the enzymes of
the heme biosynthetic pathway that are the most sensitive to lead inhibition. 5-
Aminolevulinate synthase catalyzes the rate limiting reaction in the heme
biosynthetic pathway. Glycine and succinyl CoA are combined in this initial
reaction of heme synthesis to form 5-Aminolevulinic Acid (5-ALA). In the second
reaction, catalyzed by 6-aminolevulinate dehydratase, two molecules of 5-ALA
condense to form the pyrrole porphobilinogen. In the final step of the heme
synthetic 2+
pathway, iron (Fe ) is incorporated into protoporphyrin IX in a reaction catalyzed
by ferrochelatase.
The enzymes 6-Aminolevulinate dehydratase, which contains zinc, and ferrochelatase
are inactivated by lead. Thus, in lead poisoning, 5-ALA and protoporphyrin IX
accumulate, and the production of heme is decreased leading to microcytic
hypochromic anemia secondary to a lack of hemoglobin
(Choice A) 5-Aminolevulinate synthase is the first enzyme involved in the
biosynthesis of heme. This first reaction in the heme biosynthetic pathway is the
rate-limiting step, and it utilizes pyridoxine (Vitamin Bg) as a
cofactor. Increased levels of heme inhibit this reaction.
(Choice C) Defects in uroporphyrinogen I synthase lead to acute intermittent
porphyria. These patients experience acute attacks of abdominal pain and
neuropsychiatric symptoms. They do not experience photosensitivity, but their urine
will darken upon exposure to light.
(Choice D) Defects in uroporphyrinogen decarboxylase cause the most common form of
porphyria -porphyria cutanea tarda (PCT). In patients with PCT, uroporphyrinogen
accumulates in the urine and patients experience mild photosensitivity.
(Choice E) Bilirubin glucuronyl transferase is the enzyme necessary for the
conjugation of bilirubin Either a lack of this enzyme or the presence of drugs that
interfere with glucuronyl transferase impairs the liver's ability to conjugate
bilirubin.
Educational Objective:
The zinc-containing 5-Aminolevulinate dehydratase and ferrochelatase are enzymes in
the heme biosynthetic pathway that are inactivated by lead. Thus, in lead
poisoning, 5-ALA and protoporphyrin IX accumulate, and the production of heme is
decreased, leading to microcytic anemia secondary to a lack of hemoglobin.

A protein's primary structure is the sequence of amino acids linked by covalent


peptide bonds (Choice C) Proteins may also assume a secondary structure, such as
the alpha-helix or beta-sheet, due to subsequent hydrogen bonding. In patients with
Alzheimer disease, beta-amyloid protein loses its alpha-helical configuration and
forms beta-sheets, which are less soluble and therefore prone to aggregating.
Aggregations of beta-sheets are the primary component of the extracellular senile
(neuritic) plaques found in Alzheimer patients.
The conversion of alpha-helices to beta-sheets involves the breaking and reforming
of hydrogen bonds (Choice D). Tertiary structure is the overall shape that a single
polypeptide chain assumes following compact folding of the secondary structure.
Many forces combine to stabilize the tertiary structure, including ionic bonds
(Choice A), hydrophobic interactions (Choice B). hydrogen bonds (Choice D), and
disulfide bonds (Choice E). Remember that disulfide bonds are very strong covalent
bonds between two cysteine residues within the same polypeptide chain that enhance
a protein's ability to withstand denaturation.
Educational objective:
Hydrogen bonds are the principal stabilizing force for the secondary structure of
proteins.

Tetrahydrobiopterin (BH4) is a cofactor in the synthesis of tyrosine, dopa, and


serotonin, as well as nitric oxide. Dihydrobiopterin reductase enzymatically
reduces dihydrobiopterin (BH2) to tetrabiopterin (BH4). BH4
is a cofactor used in the synthesis of tyrosine (a precursor of dopa and melanin),
dopa (an antecedent of catecholamines), and serotonin. Serotonin (5-
hydroxytryptamine, or 5HT) is formed through the hydroxylation and decarboxylation
of the amino acid tryptophan Dihydrobiopterin reductase deficiency is also
important because, while most cases of phenylketonuria are caused by a lack of
function of phenylalanine hydroxylase, a minority are actually due to
dihydrobiopterin reductase deficiency.
(Choice A) Acetylcholine is a simple molecule synthesized from choline and acetyl-
CoA by choline acetyltransferase.
(Choice C) The amino acid derivative g-aminobutyrate (GABA) is a well-known
inhibitor of presynaptic transmission in the CNS. The formation of GABA occurs via
the decarboxylation of glutamate, a reaction catalyzed by glutamate decarboxylase
(GAD).
(Choice D) The multiple roles of glutamate in nitrogen balance make it a gateway
between free ammonia and the amino groups of most amino acids; through
transamination reactions of alpha-keto acids, glutamate is able to be the principal
amino donor in the synthesis of many amino acids.
(Choice E) Glycine is an inhibitory neurotransmitter in the CNS, with particular
influence in the spinal cord. Glycine causes an inhibitory post-synaptic
hyperpolarizing potential by binding to glycine receptors and causing chloride to
enter the neuron via inotropic receptors.
Educational Objective:
BH4 is a cofactor used by hydroxylase enzymes in the synthesis of tyrosine, dopa,
and serotonin, as well as
nitric oxide. Serotonin is synthesized from tryptophan, and the initial step in
this reaction is catalyzed by an enzyme that uses BH4 as a cofactor. Enzyme
dihydrobiopterin reductase deficiency causes defective
regeneration of BH<, and is an uncommon cause of phenylketonuria (PKU).

During the process of cell division, DNA replication occurs secondary to the
coordinated effects of multiple enzymes and proteins. DNA polymerases are the
primary enzymes responsible for DNA replication, but they can not function without
the assistance of other enzymes such as primase, helicase, ligase, and
topoisomerase I and II. In E. coli, there are three primary types of DNA
polymerases: DNA polymerase I, II, and III. Primase forms the 3' OH group primer to
initiate replication of daughter strands, while helicase promotes unwinding and
dissociation of the parent strands. On the other hand, topoisomerases reduce
positive and negative supercoiling in order to relieve the strain produced by DNA
unwinding
DNA replication requires a high degree of fidelity; therefore, as synthesis of the
daughter strands proceeds, DNA polymerases proof read to ensure that the daughter
DNA is the exact complement of the parent DNA. All three prokaryotic DNA
polymerases have proof reading activity and remove mismatched nucleotides via a 3'
to 5' exonuclease activity (Choice C). Only DNA polymerase I has 5' to 3'
exonuclease activity which is used to excise and replace RNA primers and damaged
DNA sequences, which are identified by endonucleases (Choice E)
(Choice A) In contrast to exonucleases, which remove nucleotides from the end of a
DNA molecule, endonucleases cut DNA at very specific DNA sequences within the
molecule. Restriction endonucleases digest DNA into smaller fragments in a
sequence-specific manner
(Choice B) One of the major methods of DNA damage by ultraviolet light is the
dimerization of adjacent pyrimidine bases to form thymidine dimers. These dimers
are routinely formed after exposure to sunlight, but are usually removed by
protective enzymatic mechanisms.
Educational Objective:
All three prokaryotic DNA polymerases have proof reading activity and remove
mismatched nucleotides via 3' to 5' exonuclease activity. Only DNA polymerase I has
5' to 3' exonuclease activity which is used to excise and replace RNA primers and
damaged DNA sequences.

Niemann-Pick disease Type A is an autosomal recessive disorder that typically


presents in infants of Ashkenazi Jewish descent. The disease is characterized by a
deficiency of sphingomyelinase that causes sphingomyelin to accumulate within
phagocytes. The resultant "foamy histiocytes" accumulate in the liver, spleen, and
skin. Additionally, there is gradual sphingomyelin deposition in the CNS, which
causes neurologic degeneration. Affected infants classically present with loss of
previously acquired motor capabilities following a period of normal development.
The neurologic deterioration typically progresses to hypotonia and blindness by age
1. A cherry-red macular spot (as in Tay-Sachs disease) and hepatosplenomegaly are
common findings on physical examination. Death usually occurs before age 3.
(Choice A) In Tay-Sachs disease, deficiency of the enzyme hexosaminidase A causes
Gw ganglioside accumulation.
(Choice B) Hurler syndrome is a mucopolysaccharidosis where heparan sulfate and
dermatan sulfate accumulate due to a deficiency of alpha-L-iduronidase.
(Choice D) In Gaucher disease, glucocerebroside accumulates within phagocytes due
to a deficiency of glucocerebrosidase.
(Choices E and F) Von Gierke disease and Pompe disease are glycogen storage
diseases. In von Gierke disease, there is a deficiency of glucose-6-phosphatase. In
Pompe disease, there is a deficiency of lysosomal alpha-1,4-glucosidase.
(Choice G) In Fabry disease, a deficiency of alpha-galactosidase A causes the
ceramide trihexoside to accumulate. Disease manifestations include angiokeratomas,
acroparesthesia, hypohidrosis and renal failure.
(Choice H) Lesch-Nyhan syndrome is a disorder of uric acid metabolism caused by a
deficiency of the enzyme hypoxanthine guanine phosphoribosyl transferase (HGPRT).
Educational Objective:
Niemann-Pick disease is an autosomal recessive disorder characterized by a
deficiency of the sphingomyelinase enzyme and resultant accumulation of
sphingomyelin. Patients present in infancy with loss of motor skills,
hepatosplenomegaly, hypotonia and a cherry-red macular spot. Foamy histiocytes are
the classic finding on tissue histology. Death occurs before age 3.

This patient's presentation is consistent with homocystinuria, the most common


inborn error of methionine metabolism. Most patients present at age 3-10 years with
ectopia lentis (dislocated lens). About half of all patients experience some form
of intellectual disability. Other clinical manifestations include a marfanoid
habitus (eg, elongated limbs, arachnodactyly, scoliosis) and osteoporosis. Patients
are at high risk of thromboembolic episodes involving both large and small vessels,
especially those of the brain, heart, and kidneys. Thromboembolic complications are
the major cause of morbidity and mortality in these patients.
Homocystinuria is most frequently caused by deficiency of cystathionine synthetase,
an enzyme that requires pyridoxine (vitamin B6) as a cofactor. Many patients with
homocystinuria respond dramatically to pyridoxine (B6) supplementation, which
improves residual enzymatic activity and reduces plasma homocysteine levels.
(Choice A) Vitamin C is a necessary cofactor for the hydroxylation of proline and
lysine residues in collagen. Vitamin C deficiency results in decreased strength of
collagen fibers, causing a condition known as scurvy.
(Choice B) Excess dietary methionine causes an increase in plasma homocysteine
levels in patients with homocystinuria as it cannot be metabolized effectively to
cysteine As a result, treatment frequently involves restriction of dietary
methionine intake as well as cysteine supplementation.
(Choice D) Thiamine (vitamin B,) deficiency can cause dry and wet beriberi and
Wernicke-Korsakoff syndrome.
(Choice E) Tyrosine is a nonessential amino acid and the precursor for
catecholamines such as dopamine, epinephrine, and norepinephrine.
(Choice F) Vitamin K is involved in the post-translational conversion of glutamate
to gamma-carboxyglutamic acid. This modification is necessary for the function of
many clotting factors and regulatory proteins involved in the coagulation cascade.
Educational objective:
Homocystinuria is caused by cystathionine synthetase deficiency. Affected
individuals manifest with symptoms resembling those of Marfan syndrome,
particularly ectopia lentis, and many experience some form of developmental delay.
They are also at high risk of developing thromboembolism. About 50% of affected
patients respond to high doses of vitamin B6 (pyridoxine).

The infant described in the question is most likely suffering from maple syrup
urine disease, a disorder characterized by defective breakdown of the branched
chain amino acids: leucine, isoleucine, and valine. The specific defect of maple
syrup urine disease occurs in the enzyme branched chain a-keto acid dehydrogenase
Because their degradation is inhibited at the a-keto acid stage, tissue and serum
levels of these branched chain a-keto acids increase, which leads to neurotoxicity.
Maple syrup urine disease usually manifests within the first few days of life, and
classically, the urine of affected infants has a distinctive sweet odor, much like
burned caramel Maple syrup urine disease can be life threatening, if left
untreated, but dietary restriction of branched chain amino acids, such as leucine,
can lessen the severity of symptoms.
(Choice A) The amino acid phenylalanine is restricted in patients with
phenylketonuria.
(Choice B) Defective breakdown of tyrosine results in hypertyrosinemia or
alkaptonuria. A diet low in tyrosine and phenylalanine can be of benefit for the
treatment of alkaptonuria and a certain forms of hypertyrosinemia.
(Choices D, E and F) Hypermethioninemia, typically a benign disorder, results from
defective metabolism of methionine by the enzyme methionine adenosyltransferase.
Similarly, most enzymatic defects resulting in hyperlysinemia and histidinemia are
also benign disorders. Methionine restriction and cysteine supplementation is
required for treatment of classic homocystinuria.
Educational Objective:
Maple syrup urine disease (MSUD) is caused by a defect in a-keto acid
dehydrogenase, leading to an inability to degrade branched chain amino acids beyond
their deaminated a-keto acid state. This illness classically results in dystonia
and poor feeding as well as the "maple syrup scent" of the patient's urine within
the first few days of life Treatment rests on dietary restriction of branched-chain
amino acids.

Ubiquitin is a protein found in all eukaryotic cells that undergoes ATP-dependent


attachment to other proteins, labeling them for degradation The proteasome then
recognizes these ubiquinated proteins and uses ATP energy to drive them through its
tubular structure, degrading them into small peptides in the process. Attachment of
4 or more ubiquitin monomers is required before most proteins are allowed entry
into the proteasome. Ubiquitination plays an important role in many cell functions,
including antigen processing, muscle wasting, cell cycle regulation, DNA repair,
and disposal of misfolded proteins and regulatory enzymes.
Impairment of the ubiquitin-proteasome system can contribute to the development of
neurodegenerative disorders such as Parkinson's and Alzheimer's diseases. Failure
of the system to properly degrade abnormal proteins causes protein misfolding,
aggregation, and eventual obstruction of intracellular molecular traffic, leading
to cell death Together, the Parkin, PINK1, and DJ-1 genes code for a protein
complex that promotes the degradation of misfolded proteins via the ubiquitin-
proteasome system Mutations in Parkin, PINK1, and DJ-1 are each associated with
autosomal recessive forms of Parkinson's disease that have an early age of onset (<
50 years).
(Choice A) Heterochromatin is condensed and methylated DNA that has a low level of
transcriptional activity. In contrast to heterochromatin, euchromatin (loosely
arranged chromatin) has very high levels of transcriptional activity. Histone
acetylation promotes the formation of euchromatin.
(Choice B) Vitamin K-dependent gamma-carboxylation is critical for the functioning
of clotting factors II, VII,
IX, and X. and of the anticoagulative proteins C and S. Warfarin inhibits
carboxylation of these proteins
(Choice C) The hepatic processing of bilirubin is accomplished in three key steps:
carrier-mediated passive uptake of bilirubin at the sinusoidal membrane;
conjugation of bilirubin with glucuronic acid in the endoplasmic reticulum: and
active biliary excretion of the water-soluble, nontoxic bilirubin-glucuronides
Disruption of this process occurs in Crigler-Najjar syndrome, a condition in which
patients lack the enzyme needed to catalyze bilirubin glucuronidation.
(Choice D) Phosphorylation is the addition of a phosphate group (POf) to a protein
or other organic molecule. Phosphorylation is commonly involved in the regulation
of enzymatic activity.
Educational objective:
Ubiquitin is a protein that undergoes ATP-dependent attachment to other proteins,
labeling them for degradation. These modified proteins enter the proteasome and are
degraded into small peptides Impairment of the ubiquitin-proteasome system can
contribute to the development of neurodegenerative disorders, including Parkinson’s
and Alzheimer's diseases.

Alanine and glutamine play an important role in transporting nitrogen throughout


the body. Glutamine is produced by most body tissues and is catabolized primarily
by the gut and kidney for maintenance of cellular metabolism and acid-base
regulation, respectively. A significant portion of the glutamine used by these
tissues is converted to alanine and released into the circulation. Alanine is also
released by skeletal muscle tissue during protein catabolism as part of the
glucose-alanine cycle that helps remove excess nitrogen. Alanine is then
transported to the liver, where it serves as a vehicle for nitrogen disposal and as
a source of carbon skeletons for gluconeogenesis.
In the liver, alanine is transaminated by alanine aminotransferase to pyruvate with
the amino group being transferred to a-ketoglutarate to form glutamate. Almost all
aminotransferase enzymes use a-ketoglutarate as the amino group acceptor. Thus,
amino groups are tunneled into glutamate during protein catabolism. Glutamate is
further metabolized by the enzyme glutamate dehydrogenase, which liberates free
ammonia and regenerates a-ketoglutarate. Ammonia then enters the urea cycle to form
urea, the primary disposal form of nitrogen in humans. Urea subsequently enters the
blood and is excreted in the urine.
(Choices C, D, and E) Malate, citrate, and oxaloacetate are all intermediates of
the tricarboxylic acid cycle.
(Choice B) L-citrulline is an amino acid produced as an intermediate in the
conversion of ornithine to argininosuccinate during the hepatic urea cycle.
Educational objective:
Alanine is the major amino acid responsible for transferring nitrogen to the liver
for disposal. During the catabolism of proteins, amino groups are transferred to a-
ketoglutarate to form glutamate. Glutamate is then processed in the liver to form
urea, the primary disposal form of nitrogen in humans Free ammonia is also excreted
into the urine by the kidney for regulation of acid-base status.

Orotic aciduria is a disorder of pyrimidine metabolism characterized by hypochromic


megaloblastic anemia, neurologic abnormalities, growth retardation and excretion of
high amounts of orotic acid in the urine Pyrimidine synthesis begins with formation
of carbamoyl phosphate from ATP, CO, and glutamine by carbamoyl phosphate
synthetase II (CPS-II) This is the regulatory step for pyrimidine synthesis.
Carbamoyl phosphate is subsequently converted to orotate by a series of reactions.
The defective enzymes in orotic aciduria are orotate phosphoribosyl transferase and
OMP decarboxylase These two enzymes represent separate active domains on a single
polypeptide; this is why a single mutation causes dysfunction of both enzymes.
These enzymes catalyze the final conversion of orotate to uridine 5'-monophosphate
(UMP). This disorder is treated with uridine supplementation. The supplemental
uridine is converted to UMP by the action of nucleoside kinases, and UMP in turn
inhibits CPS-II, thus attenuating orotic acid production
(Choice A) Folate participates in single carbon transfer reactions, as in the de
novo synthesis of purine and thymidine. Folate supplements will improve
megaloblastic anemia resulting from folate deficiency but it will not improve the
anemia in orotic aciduria.
(Choice B) Iron supplementation improves iron deficiency anemia, classically a
microcytic hypochromic anemia.
(Choice C) Pyridoxine (vitamin B6) supplementation is indicated during treatment
with isoniazid. Pyridoxine is a cofactor in transamination, deamination,
decarboxylation and condensation reactions.
(Choice D) Ascorbic acid (vitamin C) is required for hydroxylation of proline and
lysine residues in collagen synthesis; therefore, it plays an important role in
connective tissue maintenance and wound healing.
(Choice E) Guanine and adenine are purine bases present in DNA and RNA. Orotic
aciduria is a defect in the synthesis of pyrimidine bases, so supplementation with
purines would not affect the synthesis of orotate.
Educational objective:
A patient with orotic aciduria (impaired de novo pyrimidine synthesis) will present
with hypochromic megaloblastic anemia, neurologic abnormalities, growth retardation
and excretion of orotic acid in the urine. Uridine supplementation improves
symptoms by inhibiting carbamoyl phosphate synthetase II.

This patient's severe intellectual disability, history of seizures, and abnormal


pallor of catecholaminergic brain nuclei on autopsy are suggestive of
phenylketonuria (PKU). PKU results from the inability to convert phenylalanine into
tyrosine, a reaction which is normally catalyzed by phenylalanine hydroxylase. This
enzyme requires the cofactor tetrahydrobiopterin (BH4), which is regenerated from
dihydrobiopterin (BH2) by the enzyme dihydropteridine reductase. Although neonatal
hyperphenylalaninemia can be caused by deficiency of either enzyme, most cases are
attributable to abnormalities in phenylalanine hydroxylase.
It is believed that excess phenylalanine and the presence of large concentrations
of phenylalanine metabolites (eg, phenyllactate & phenylacetate) contribute to the
brain damage seen in PKU. Hypopigmentation involving the skin, hair, eyes, and
catecholaminergic brain nuclei (which produce a dark pigment known as neuromelanin)
results from the inhibitory effect of excess phenylalanine on melanin synthesis.
The classic musty or mousy body odor is due to the accumulation of abnormal
phenylalanine metabolites.
(Choice A) Branched-chain ketoacid dehydrogenase catalyzes decarboxylation of the
a-ketoacid derivatives of all 3 branched chain amino acids: leucine, isoleucine,
and valine. Deficiency causes maple syrup urine disease, which is characterized by
a burnt-sugar smell in the urine of affected patients Hypopigmentation is not seen
in the condition
(Choice B) Dopamine hydroxylase is the enzyme that catalyzes the biosynthesis of
norepinephrine from dopamine. Deficiency of this enzyme causes a rare form of
dysautonomia characterized by ptosis, orthostatic hypotension, hypoglycemia, and
hypothermia.
(Choice C) Alkaptonuria is an autosomal recessive disorder of tyrosine degradation
caused by a deficiency of homogentisic acid oxidase. This condition results in the
accumulation of large amounts of homogentisic acid, leading to connective tissue
hyperpigmentation and degenerative joint disease.
(Choice E) Albinism is an autosomal recessive disorder caused by defects in the
biosynthesis and distribution of melanin. It is most commonly due to depressed or
absent tyrosinase activity. Melanin synthesis begins with the conversion of
tyrosine to DOPA and DOPA to dopaquinone by the enzyme tyrosinase in melanocytes.
Albinism does not present with neurologic dysfunction.
Educational objective:
Deficiency of the enzyme phenylalanine hydroxylase or its cofactor
tetrahydrobiopterin causes accumulation of phenylalanine in body fluids and the
central nervous system. Homozygous infants are normal at birth but gradually
develop severe intellectual disability and seizures if left untreated.
Hypopigmentation of the skin, hair eves, and catecholamineraic brain nuclei is also
freauentlv seen.

This patient has Lesch-Nyhan syndrome, an X-linked recessive disorder of purine


metabolism. Normally, the free purine bases (adenine, guanine, and hypoxanthine)
generated during cellular metabolism of nucleic acids are reconverted into their
corresponding nucleotides by phosphoribosylation in the purine salvage pathway
Hypoxanthine-guanine phosphoribosyltransferase (HGPRT) is the main enzyme of this
pathway, converting hypoxanthine to inosine monophosphate and guanine to guanosine
monophosphate.
In Lesch-Nyhan syndrome there is a deficiency of HGPRT, which results in failure of
purine salvage This results in increased degradation of guanine and hypoxanthine
bases into uric acid, thereby causing hyperuricemia. Excessive levels of
phosphoribosyl-pyrophosphate (PRPP) also develop, as PRPP is a substrate used by
HGPRT during purine salvage and it accumulates when there is deficiency of the
enzyme. PRPP is created by PRPP synthetase during the first step of de novo purine
synthesis. The increased PRPP concentrations present in Lesch-Nyhan syndrome
increase the activity of the downstream enzymes involved in de novo purine
biosynthesis.
Phosphoribosyl pyrophosphate amidotransferase acts upon PRPP to form
phosphoribosylamine in the first committed step of de novo purine synthesis;
therefore, its activity will be increased in Lesch-Nyhan syndrome secondary to the
increased PRPP concentrations (Choice C) Note that the final steps in de novo
purine synthesis (conversion of inosine monophosphate to either AMP or GMP) require
ATP for GMP formation and GTP for AMP formation. This cross-regulation decreases
the synthesis of one nucleotide when the other is deficient (thus shunting
conversion of IMP toward the deficient nucleotide).
(Choices A, D, and E) These enzymes participate in pyrimidine synthesis.
(Choice B) Hypoxanthine-guanine phosphoribosyltransferase is the defective enzyme
in Lesch-Nyhan syndrome; its activity is decreased in this condition.
(Choice F) Purine and pyrimidine nucleosides initially contain a ribose sugar The
enzyme ribonucleotide reductase converts ribose sugars to their deoxyribose forms
(ADP, GDP, CDP, and UDP to dADP, dGDP, dCDP, and dUDP, respectively)
Educational objective:
Lesch-Nyhan syndrome is an X-linked recessive disorder caused by a defect in
hypoxanthine-guanine phosphoribosyltransferase (HGPRT). This results in failure of
the purine salvage pathway. Because they are not recycled, increased amounts of the
purine bases hypoxanthine and guanine are degraded to uric acid De novo purine
synthesis must increase to replace the lost bases.

This infant is most likely suffering from maple syrup urine disease (MSUD), a
disorder characterized by the defective breakdown of branched chain amino acids
(leucine, isoleucine, and valine). Degradation of these amino acids first involves
transamination to their respective a-ketoacids, which are subsequently metabolized
by an enzyme complex referred to as branched-chain a-ketoacid dehydrogenase. MSUD
can result from mutations in any of the 4 genes coding for the 3 catalytic subunits
of this complex. Neurotoxicity results primarily from the accumulation of leucine
in the serum and tissues. A metabolite of isoleucine gives the urine of affected
infants a distinctive sweet odor much like burned caramel. MSUD can be life-
threatening if untreated, but dietary restriction of branched chain amino acids can
lessen the severity of symptoms.
Branched-chain a-ketoacid dehydrogenase, pyruvate dehydrogenase, and a-
ketoglutarate dehydrogenase all require five cofactors: Thiamine pyrophosphate,
Lipoate, Coenzyme A, FAD, NAD (mnemonic: Tender Loving Care For Nancy). Some
patients with MSUD improve with high-dose thiamine treatment (thiamine-responsive),
but most still require lifelong dietary restrictions.
(Choice A) Galactocerebrosidase catalyzes the liposomal hydrolysis of
galactocerebroside, a galactolipid that is found in abundance in myelin. Krabbe
disease (globoid cell leukodystrophy) is a rare autosomal recessive disorder caused
by deficiency of this enzyme. The infantile form of this disease typically
manifests between 2-5 months of age with irritability, developmental delay or
regression, and muscle tone abnormalities.
(Choice B) Pyridoxine (vitamin B6) as pyridoxal phosphate is involved in the
transamination and decarboxylation steps in amino acid metabolism as well as heme
and neurotransmitter synthesis. Pyridoxine supplementation is used in the treatment
of sideroblastic anemia and hyperhomocysteinemia
(Choice D) Folic acid, frequently deficient in alcoholics, is responsible for the
transfer of single carbon moieties during nucleic acid synthesis. Folic acid
supplementation is used for the treatment of hyperhomocysteinemia and in the
prevention of neural tube defects in newborns.
(Choices E and F) A variant of phenylketonuria is due to deficiency of
tetrahydrobiopterin, a cofactor for phenylalanine hydroxylase, the enzyme that
converts phenylalanine to tyrosine. Tetrahydrobiopterin supplementation can reduce
phenylalanine levels in these particular patients.
Educational objective:
Branched-chain a-ketoacid dehydrogenase, similar to pyruvate and a-ketoglutarate
dehydrogenase, requires several coenzymes: Thiamine pyrophosphate, Lipoate,
Coenzyme A, FAD, NAD (mnemonic: Tender Loving Care For Nancy). Some patients with
maple syrup urine disease improve with high-dose thiamine treatment (thiamine-
responsive), but most still require lifelong dietary restrictions.

Wernicke syndrome manifests with the triad of ophthalmoplegia, ataxia, and


confusion It is lethal in 10-20% of patients. Foci of hemorrhage and necrosis in
the mamillary bodies and periaqueductal gray matter are found on autopsy. This
condition occurs due to chronic thiamine deficiency, a condition common in patients
with alcoholism.
Thiamine (vitamin B1) participates in a number of reactions of glucose metabolism.
It is a cofactor for the following enzymes:
1. Pyruvate dehydrogenase converts pyruvate (the end-product of glycolysis) into
acetyl CoA (which enters the citric acid cycle).
2 a-ketoglutarate dehydrogenase is an enzyme of the citric acid cycle.
3. Transketolase is an enzyme of the hexose monophosphate pathway. It converts
pentoses (derived from glucose) to glyceraldehyde 3P (an intermediary of
glycolysis).
Thiamine deficiency, therefore, results in decreased glucose utilization, which is
especially pronounced in the CNS. If a patient with chronic thiamine deficiency is
given a glucose infusion without thiamine supplementation, acute cerebral damage
occurs An increase in erythrocyte transketolase levels after thiamine infusion is
diagnostic for thiamine deficiency. (In actual practice, if a patient might be an
alcoholic or appears to be very malnourished, presume that the patient is thiamine
deficient and give thiamine supplementation with glucose infusion.)
(Choices B and D) Neither erythrocyte glutathione reductase, nor NAD are used for
the diagnosis of thiamine deficiency.
(Choice C) Erythrocyte glucose-6-phosphate dehydrogenase (G6PD) catalyzes a rate-
limiting step in the pentose phosphate pathway. This pathway is necessary for NADPH
production and the function of the erythrocyte antioxidant system Decreased
glucose-6-phosphate dehydrogenase levels cause hemolytic anemia
(Choice E) Methylmalonic acid is a product of fatty acid oxidation It is converted
to succinyl CoA by methylmalonyl CoA mutase. This enzyme uses B12 as a coenzyme.
Methylmalonic acid levels are increased in vitamin B12 deficiency.
(Choice F) Protoporphyrin is one of the precursors of heme. An increased
erythrocyte protoporphyrin concentration is the hallmark of erythropoietic
protoporphyria (EPP); however, this elevation is nonspecific and can be seen in
other conditions such as iron-deficient anemia and lead poisoning.

Ammonia generated from the metabolism of alpha amino acids is converted into urea
by the urea cycle. The urea cycle involves five enzymatic steps, two in
mitochondrial matrix, and three in the cytosol. The combination of CO., ammonia,
and ATP, catalyzed by carbamoyl phosphate synthase (the rate-limiting enzyme in the
urea cycle), forms carbamoyl phosphate as the first step of the urea cycle.
Carbamoyl phosphate then combines with ornithine to form citrulline in a reaction
catalyzed by ornithine transcarbamoylase in the mitochondrial matrix. Citrulline
then enters the cytosol and is converted to argininosuccinate, which is then
converted to arginine. The conversion of arginine to ornithine by the cytosolic
enzyme arginase completes the urea cycle by releasing a urea molecule
Remember that urea synthesis is a cyclic process and that while ammonium ion, CO.,
ATP, and aspartate are consumed in this process, there is no net loss or gain of
ornithine, citrulline, argininosuccinate, or arginine. The molecule N-
acetylglutamate serves as a regulator of the urea cycle through allosteric
activation of carbamoyl phosphate synthetase I.
Disorders of the urea cycle can result from defects in any of the following six
enzymes:
1. Carbamoyl phosphate synthetase (CPS)
2. Ornithine transcarbamoylase (OTC)
3. Argininosuccinic acid synthetase (AS)
4. Argininosuccinic acid lyase (AL)
5. Arginase (AG)
6. N-Acetylglutamate synthetase (NAGS)
The first five enzymes are directly involved in the urea cycle whereas the sixth
enzyme is involved in the production of N-acetylglutamate, the allosteric activator
of carbamoyl phosphate synthase I. Patients with urea cycle disorders display
clinical symptoms of neurological damage secondary to increased serum ammonia
levels. Typically, patients present early in childhood, however, milder defects can
present for the first time during adulthood. OTC deficiency is the most common urea
cycle disorder, resulting in increased levels of carbamoyl phosphate and impaired
disposal of ammonia
(Choices A and C) CPS and NAGS defects result in increased blood levels of ammonia
and neurological disorders, but low levels of carbamoyl phosphate and no elevation
in urinary orotic acid (Remember that in OTC deficiency, accumulated carbamoyl
phosphate is converted into orotic acid).
(Choice D) Deficiency of HPRT (Lesch-Nyhan syndrome) results in excessive uric acid
production because purines cannot be salvaged from degraded DNA. The clinical
manifestations of HPRT deficiency include hyperuricemia, urate kidney stones, self-
mutilation, and involuntary movements,
(Choice E) Adenosine deaminase (ADA) is an enzyme involved in purine metabolism.
Decreased expression of ADA causes severe combined immune deficiency (SCID) as the
accumulation of adenosine is toxic to lymphocytes.
Educational Objective:
Ornithine transcarbamoylase deficiency is the most common disorder of the urea
cycle, resulting in severe neurological abnormalities due to high blood and tissue
ammonia levels. Increased urine orotic acid excretion is typical.

(Choice D) Deficiency of HPRT (Lesch-Nyhan syndrome) results in excessive uric acid


production because purines cannot be salvaged from degraded DNA. The clinical
manifestations of HPRT deficiency include hyperuricemia, urate kidney stones, self-
mutilation, and involuntary movements,
(Choice E) Adenosine deaminase (ADA) is an enzyme involved in purine metabolism.
Decreased expression of ADA causes severe combined immune deficiency (SCID) as the
accumulation of adenosine is toxic to lymphocytes.
Educational Objective:
Ornithine transcarbamoylase deficiency is the most common disorder of the urea
cycle, resulting in severe neurological abnormalities due to high blood and tissue
ammonia levels. Increased urine orotic acid excretion is typical.
(Choice C) Proteasomes function to degrade unneeded or improperly formed
intracellular proteins to small polypeptides or to amino acids. The proteasome is
essential for regulation of cellular processes because it degrades proteins that
express a function that is no longer needed by the cell. Proteasomes also function
to degrade viral proteins for expression on MHC Class I molecules for recognition
by T lymphocytes.
(Choice D) Lysosomes are organelles containing an acidic fluid with various
proteins for degrading fatty acids, carbohydrates, proteins and nucleic acids.
Diseases resulting from lysosomal dysfunction include the mucopolysaccharidoses
most classically among others.
(Choice F) The Golgi apparatus serves to sort proteins from the rough ER and route
them to their ultimate location within membrane-bound vesicles
Educational Objective:
Peroxisomal diseases are rare inborn errors of metabolism where peroxisomes are
either absent or nonfunctional. Very long chain fatty acids or fatty acids with
branch points at odd-numbered carbons can not undergo mitochondrial beta-oxidation;
these fatty acids are metabolized by a special form of beta oxidation (very long
chain fatty acids) or by alpha oxidation (branched chain fatty acids such as
phytanic acid) within peroxisomes. These diseases commonly lead to neurologic
defects from improper CNS myelination.

The patient described in the question stem exhibits features of arginase


deficiency. Arginase is an enzyme of the urea cycle that produces urea and
ornithine from arginine. Arginase deficiency is likely underdiagnosed because the
spasticity seen commonly in this disorder may simply be attributed to cerebral
palsy. Treatment of arginase deficiency consists of a low-protein diet that is
devoid of arginine. Administration of a synthetic protein made of essential amino
acids usually results in a dramatic decrease in plasma arginine concentration and
an improvement in neurologic abnormalities.
(Choice A) Serotonin (5-hydroxytryptamine, 5HT) is formed by the hydroxylation and
decarboxylation of tryptophan by tryptophan hydroxylase It is degraded by monoamine
oxidase and also undergoes neuronal reuptake
(Choice B) Glutamine is one of the 20 major amino acids. Glutamine is the major
amino acid found in the blood because it transports ammonia from peripheral tissues
to the kidney. In the nephron, the amide nitrogen is hydrolyzed by the enzyme
glutaminase to regenerate glutamate and a free ammonium ion, which can then be
excreted in the urine.
(Choice C) Orotic acid is a chemical overproduced from carbamoyl phosphate by
carbamoyl phosphate synthetase II (CPS II) in an alternative pathway when there is
a block in the urea cycle. Excessive amounts of orotic acid are usually found in
OTC (ornithine transcarbamylase) deficiency, citrullinemia, and often in
argininosuccinic aciduria.
(Choice D) Deficiencies of vitamins Be, B12, and folate are associated with high
levels of plasma homocysteine, which in turn is associated with atherosclerosis and
thrombotic events.
(Choice E) The amino acid derivative y-aminobutyrate, also called GABA, is a well-
known inhibitor of presynaptic transmission in the CNS and in the retina. The
formation of GABA occurs by the decarboxylation of glutamate catalyzed by glutamate
decarboxylase (GAD).
Educational Objective:
Arginase is an enzyme of the urea cycle that produces urea and ornithine from
arginine.

The patient described in the question stem exhibits features of arginase


deficiency. Arginase is an enzyme of the urea cycle that produces urea and
ornithine from arginine. Arginase deficiency is likely underdiagnosed because the
spasticity seen commonly in this disorder may simply be attributed to cerebral
palsy. Treatment of arginase deficiency consists of a low-protein diet that is
devoid of arginine. Administration of a synthetic protein made of essential amino
acids usually results in a dramatic decrease in plasma arginine concentration and
an improvement in neurologic abnormalities.
(Choice A) Serotonin (5-hydroxytryptamine, 5HT) is formed by the hydroxylation and
decarboxylation of tryptophan by tryptophan hydroxylase It is degraded by monoamine
oxidase and also undergoes neuronal reuptake
(Choice B) Glutamine is one of the 20 major amino acids. Glutamine is the major
amino acid found in the blood because it transports ammonia from peripheral tissues
to the kidney. In the nephron, the amide nitrogen is hydrolyzed by the enzyme
glutaminase to regenerate glutamate and a free ammonium ion, which can then be
excreted in the urine.
(Choice C) Orotic acid is a chemical overproduced from carbamoyl phosphate by
carbamoyl phosphate synthetase II (CPS II) in an alternative pathway when there is
a block in the urea cycle. Excessive amounts of orotic acid are usually found in
OTC (ornithine transcarbamylase) deficiency, citrullinemia, and often in
argininosuccinic aciduria.
(Choice D) Deficiencies of vitamins Be, B12, and folate are associated with high
levels of plasma homocysteine, which in turn is associated with atherosclerosis and
thrombotic events.
(Choice E) The amino acid derivative y-aminobutyrate, also called GABA, is a well-
known inhibitor of presynaptic transmission in the CNS and in the retina. The
formation of GABA occurs by the decarboxylation of glutamate catalyzed by glutamate
decarboxylase (GAD).
Educational Objective:
Arginase is an enzyme of the urea cycle that produces urea and ornithine from
arginine.

Catabolism of isoleucine, valine, threonine, methionine, cholesterol, and odd-chain


fatty acids leads to formation of propionic acid, which is then converted to
methylmalonic acid by biotin-dependent carboxylatiom Isomerization of methylmalonyl
CoA forms succinyl CoA, which subsequently enters the TCA cycle Defects in this
isomerization reaction lead to the development of methylmalonic acidemia
(Choice A) Hydrolysis implies lysis of a chemical compound by water. Hydrolysis is
an important reaction in carbohydrate, protein, and fat metabolism In protein
metabolism, a well-known hydrolysis reaction is the conversion of arginine to
ornithine by the enzyme arginase (in the urea cycle).
(Choice B) The introduction of a carboxyl group into a compound or molecule is
called carboxylation. The conversion of propionic acid to methylmalonic acid is
accomplished by biotin dependent carboxylation.
(Choice C) Hydroxylation is a reaction catalyzed by enzymes called hydroxylases,
which introduce one or more hydroxyl groups (-OH) into a molecule. A classic
example occurs during collagen post-translational modification, where proline and
lysine residues are hydroxylated by prolyl and lysyl hydroxylases, respectively.
(Choice E) Transamination is an important step in amino acid metabolism
Transamination converts pairs of alpha-ketoacids and amino acids by transfering the
amino group from one amino acid to an alpha-keto acid, thereby converting the
original amino acid into its alpha-keto acid and converting the original alpha-keto
acid into its amino acid. Vitamin Bg is required for most all transamination
reactions.
Educational Objective:
Methylmalonic acidemia (also known as methylmalonic aciduria) results from a defect
in the isomerization reaction that transforms methylmalonyl CoA to succinyl CoA,
prior to succinyl CoA entering the TCA Cycle.

The above image illustrates the biochemical reaction whereby phenylalanine is


converted to tyrosine by phenylalanine hydroxylase using tetrahydrobiopterin (BH,)
as a cofactor. Tyrosine is a non-essential amino acid that becomes essential in the
setting of phenylketonuria (PKU). The next step in this pathway is the conversion
of tyrosine to DOPA, catalyzed by the enzyme tyrosine hydroxylase, which also uses
BH, as a cofactor. When there is deficiency of dihydrobiopterin reductase, this
reaction is compromised (the phenylalanine hydroxylase reaction is also
compromised, but the effects are unseen in this case because the patient was
receiving tyrosine supplementation, thus bypassing the impaired phenylalanine
hydroxylase reaction). Once DOPA is synthesized, it is decarboxylated to dopamine
by the enzyme DOPA decarboxylase. Dopamine ultimately serves as the precursor
molecule to the catecholamines epinephrine and norepinephrine.
Deficiency of dihydrobiopterin reductase, the enzyme responsible for reduction of
dihydrobiopterin (BH„) to BH„ is the most common cause for a deficiency of BH,.
This results in what is known as atypical or malignant phenylketonuria.
Tetrahydrobiopterin is a cofactor for enzymes that participate in the synthesis of
tyrosine, DOPA, serotonin, and nitric oxide. Under normal conditions, dopamine from
the tuberoinfundibular system tonically inhibits the release of prolactin.
Decreased BH, causes decreased levels of dopamine, which therefore cause increased
levels of prolactin.
(Choice A) Phenylalanine hydroxylase is an enzyme that converts ingested
phenylalanine to tyrosine. The above patient's phenylalanine levels have normalized
with diet therapy, but he has low dopamine levels. Patients with classic
phenylketonuria, or phenylalanine hydroxylase deficiency, do not have any metabolic
errors producing dopamine when their diet contains adequate amounts of tyrosine.
Therefore, this patient must have dihydrobiopterin reductase deficiency, which,
even with phenylalanine restriction and tyrosine supplementation, causes low
dopamine levels (see diagram).
(Choice C) Dopamine hydroxylase is the enzyme that catalyzes the biosynthesis of
norepinephrine from dopamine. It does not use BH, as a cofactor.
(Choice D) Alkaptonuria is an autosomal-recessive disorder caused by a deficiency
of the enzyme homogentisic acid oxidase, which normally breaks down homogentisic
acid (also called alkapton), a toxic tyrosine byproduct that is harmful to bones
and cartilage.
(Choice E) Phenylethanolamine N-methyltransferase (PNMT) converts norepinephrine
into epinephrine. It requires S-adenosyl-methionine (SAM) as a cofactor.
(Choice F) Albinism is caused by defects in the biosynthesis and distribution of
melanin. Melanin is synthesized in melanocytes from tyrosine by the enzyme
tyrosinase
Educational objective:
Tetrahydrobiopterin (BHJ is a cofactor used in the synthesis of tyrosine, DOPA,
serotonin, and nitric oxide. Initially, tyrosine is converted to DOPA by the enzyme
tyrosine hydroxylase, with BHj used as a cofactor.
Next, DOPA is decarboxylated to dopamine by the enzyme DOPA decarboxylase. In
atypical phenylketonuria (PKU) with tyrosine supplementation, only the
catecholamine synthesis reactions downstream of tyrosine are compromised.

In the process of gluconeogenesis, glucose is formed from lactate, glycerol, and


glucogenic amino acids.
This process utilizes many of the enzymes involved in glycolysis. However, three of
the ten glycolytic enzymes are unidirectional because of thermodynamic barriers.
These enzymes are hexokinase, phosphofructokinase, and pyruvate kinase. Four
distinct enzymes are used to bypass these three glycolytic enzymes during the
gluconeogenic conversion of pyruvate to glucose.
To convert pyruvate to phosphoenolpyruvate in the first step of gluconeogenesis
(bypassing pyruvate kinase), pyruvate is first converted to oxaloacetate through
biotin-dependent carboxylation by the enzyme pyruvate carboxylase in the
mitochondria. The activity of pyruvate carboxylase is increased by high
concentrations of acetyl CoA. Oxaloacetate is then converted to malate by
mitochondrial malate dehydrogenase to facilitate exit from the mitochondria; in the
cytosol, it is converted back to oxaloacetate by cytosolic malate dehydrogenase
(the malate shuttle). Oxaloacetate is then converted to phosphoenolpyruvate by the
enzyme phosphoenolpyruvate carboxykinase (PEPCK). The two other differing enzymes
in gluconeogenesis are fructose 1,6-bisphosphatase (bypassing phosphofructokinase)
and glucose-6-phosphatase (bypassing hexokinase).
(Choice A) The first step involved in degradation of glycogen is breakage of 1 -4
glycosidic linkage to form glucose-1-phosphate. In the case described in the
vignette, after 24 hours of fasting, maintenance of glucose levels is achieved
through the process of gluconeogenesis, not by glycogenolysis.
(Choice B) Palmitic acid is the first fatty acid produced from acetyl CoA during
lipogenesis in the fed state During prolonged fasting, lipolysis leads to
generation of glycerol and fatty acids.
(Choice D) Conversion of fructose 6-phosphate to fructose 1,6-bisphosphate is
catalyzed during glycolysis by the unidirectional enzyme phosphofructokinase.
(Choice E) Conversion of acetoacetyl CoA to 3-hydroxy-3-methylglutaryl-coenzyme A
(HMG CoA) occurs in the synthesis of cholesterol and ketones.
Educational objective:
After 12 to 18 hours of fasting, gluconeogenesis is the principal source of blood
glucose. Gluconeogenesis uses many of the bidirectional enzymes involved in the
process of glycolysis, but a few unidirectional enzymes need to be bypassed The
initial committed step of gluconeogenesis involves the conversion of pyruvate to
oxaloacetate, and oxaloacetate to phosphoenolpyruvate.

Gluconeogenesis is not simply the reverse of glycolysis, as three of the ten


enzymes in glycolysis are unidirectional. These enzymes are hexokinase
(glucokinase), phosphofructokinase-1 (PFK-1), and pyruvate kinase. To reverse
glycolysis (to form glucose from pyruvate), four different enzymes are required to
overcome these unidirectional roadblocks. These four gluconeogenic enzymes are
pyruvate carboxylase, phosphoenolpyruvate carboxykinase, fructose 1,6-
bisphosphatase, and glucose-6-phosphatase. Fructose
2.6- bisphosphate helps control the balance between gluconeogenesis and glycolysis
through inverse regulation of phosphofructokinase-1 and fructose 1,6-
bisphosphatase.
The conversion of fructose 1,6-bisphosphate to fructose-6-phosphate occurs in
gluconeogenesis and is catalyzed by the enzyme fructose-1,6-bisphosphatase. This
enzyme is allosterically inhibited by fructose
2.6- bisphosphate Conversely, fructose 2,6-bisphosphate allosterically activates
phosphofructokinase-1, the main regulatory enzyme involved in glycolysis, which
converts fructose 6-phosphate to fructose
1.6- bisphosphate. Therefore, high concentrations of fructose 2,6-bisphosphate
inhibit gluconeogenesis and promote glycolysis.
By inhibiting gluconeogenesis, high concentrations of fructose 2,6-bisphosphate
will also lead to a decrease in the conversion of alanine to glucose (which begins
with the transamination of alanine to pyruvate). This is because gluconeogenesis
utilizes pyruvate derived from alanine as a gluconeogenic substrate, and processes
that inhibit gluconeogenesis also downregulate the production of gluconeogenic
substrates.
The interconversion of fructose-6-phosphate and fructose 2,6-bisphosphate is
achieved by a bifunctional enzyme composed of phosphofructokinase-2 (PFK-2, which
converts fructose-6-phosphate to fructose
2.6- bisphosphate) and fructose 2,6-bisphosphosphatase (which converts fructose
2,6-bisphosphate to fructose-6-phosphate). Glucagon causes phosphorylation of this
enzyme, leading to inactivation of the kinase and activation of the phosphatase.
Insulin causes dephosphorylation leading to enzymatic reversal (i.e., activation of
the kinase part of this bifunctional enzyme). Thus, insulin increases the level of
fructose
2.6- bisphosphate causing allosteric upregulation of the enzyme
phosphofructokinase-1 and augmentation of glycolysis.
(Choice A) Elevation of both insulin levels as well as glucose concentrations is
required to stimulate the formation of glycogen within the liver. As stated above,
elevated insulin levels also increase the formation of fructose 2,6-bisphosphate.
Thus, insulin-mediated hepatic glycogen synthesis is often concurrent with an
increase in fructose 2,6-bisphosphate formation
(Choice B) Fatty acid synthesis is upregulated by insulin and high citrate levels
(which increase when acetyl-CoA is abundant, as with active glycolysis). Thus,
fatty acid synthesis is likely to be upregulated in metabolic states where fructose
2,6-bisphosphate concentration is increased.
(Choice D) The conversion of fructose-6-phosphate to fructose-1,6-bisphosphate is
catalyzed by the enzyme phosphofructokinase-1. This enzyme is allosterically
activated by high levels of fructose-2,6-bisphosphate, thus conversion would be
increased.
(Choice E) Transamination reactions are not altered by the level of fructose 2,6
bisphosphate.
Educational objective:
Fructose 2,6-bisphosphate activates glycolysis by inducing phosphofructokinase-1
and inhibits gluconeogenesis by inhibiting fructose 1,6-bisphosphatase. High
concentrations of fructose 2,6-bisphosphate also decreases the gluconeogenic
conversion of alanine to glucose. Fructose 2,6-bisphosphate concentration is
regulated by a bifunctional enzyme composed of phosphofructokinase-2 and fructose
2,6-bisphosphatase.

The clinical features of the patient described in this clinical vignette are
consistent with Turner syndrome In contrast to normal females, patients with Turner
syndrome have only one X-chromosome (karyotype 45, XO). One X-chromosome in normal
46, XX females is normally randomly deactivated by the process of lyonization to
form condensed heterochromatin, which is easily identified as condensed body at the
periphery of the nucleus on microscopy, known also as the Barr body. Thus, all
human females are mosaics at the X-chromosome, which is accomplished through DIMA
methylation, as cytosine residues are converted to methylcytosine residues.
Heterochromatin is condensed and methylated DNAthat has a low level of
transcriptional activity (Choice B). In contrast to heterochromatin, euchromatin
(loosely arranged chromatin) has very high levels of transcriptional activity.
Histone acetylation is responsible for the formation of euchromatin (Choice A). In
fact, methylation of DNA forms heterochromatin with low transcriptional activity,
while histone acetylation forms euchromatin with high transcriptional activity.
(Choice C) Repair of mismatched bases occurs during DNA replication and is carried
out by DNA polymerases. Impaired mismatch repair is associated with hereditary non-
polyposis colorectal cancer.
(Choice D) During DNA replication, positive supercoiling appears in the region
ahead of the replication fork. If positive supercoiling is not removed, DNA
replication is stopped. Topoisomerases are the enzymes responsible for reducing DNA
supercoiling by nicking the DNA strands, introducing negative coiling, and
religating the strands.
(Choice E) Compared to single strand breakage, double strand DNA breakage is
difficult to repair and occurs following exposure to ionizing radiation Double
stranded break repair is more prone to result in faulty repair, leading to
mutations, malignancy, or cell death.
Educational Objective:
Heavily methylated DNA is typically found in heterochromatin, which is condensed
and transcriptionally inactive. Tight association with non-acetylated histones and
methylation both contribute to the compact nature of heterochromatin and its
transcriptionally inactive state.

Causes of lactic acidosis


• Enhanced metabolic rate (eg. seizures and exercise)
9 Reduced oxygen delivery (eg, cardiac or pulmonary failure, shock, and tissue
infarction)
• Diminished lactate catabolism due to hepatic failure or hypoperfusion
• Decreased oxygen utilization (eg, cyanide poisoning)
• Enzymatic defects in glycogenolysis or gluconeogenesis
This patient's signs and symptoms (eg, fever, leukocytosis, hypotension, and
tachycardia) suggest that he is in septic shock His decreased bicarbonate level and
increased anion gap are indicative of anion-gap metabolic acidosis, and his
elevated lactic acid level suggests that these metabolic derangements are secondary
to lactic acidosis.
Lactic acidosis is an anion-gap metabolic acidosis that results from overproduction
and/or impaired clearance of lactic acid. In septic shock, impaired tissue
oxygenation decreases oxidative phosphorylation, leading to the shunting of
pyruvate to lactate after glycolysis. Hence, there is an increase in lactic acid
formation. Hepatic hypoperfusion also contributes to the buildup of lactic acid, as
the liver is the primary site of lactate clearance
(Choice A) Impaired hepatic gluconeogenesis can result in the buildup of lactic
acid, as seen in congenital pyruvate carboxylase deficiency. However, this
patient's serum glucose level, lack of specific history suggestive of enzymatic
deficiency, and symptoms of septic shock make this option unlikely.
(Choice B) Increased lipolysis and ketogenesis occur in patients with diabetic
ketoacidosis, which also presents with an anion-gap metabolic acidosis.
(Choice C) Increased protein breakdown can occur in the setting of chronic
metabolic acidosis. However, it is not a direct cause of acidosis.
(Choice D) Impaired renal tubular bicarbonate reabsorption produces type 2
(proximal) renal tubular acidosis. Poor bicarbonate reabsorption can occur in a
variety of inherited or acquired conditions, including multiple myeloma and drug
toxicity (eg, acetazolamide).
Educational objective:
Lactic acidosis occurs in patients with septic shock because of tissue hypoxia,
which results in impaired oxidative phosphorylation and the shunting of pyruvate to
lactate following glycolysis. Hepatic hypoperfusion also contributes to the buildup
of lactic acid, as the liver is the primary site of lactate clearance.
Predisposing conditions for aspiration pneumonia
• Altered consciousness impairing cough reflex and glottic closure (eg.
dementia and drug intoxication)
• Dysphagia due to neurologic deficits (eg, stroke and neurodegenerative
disease)
• Upper gastrointestinal tract disorders (eg, GERD)
• Mechanical compromise of aspiration defenses (eg. nasogastric and
endotracheal tubes)
• Protracted vomiting
• Large-volume tube feedings in recumbent position
This patient's fever, leukocytosis, and radiographic lung opacities are most
consistent with pneumonia. Given his history of dementia and stroke with residual
hemiparesis, the pneumonia is most likely due to aspiration. The chest x-ray
findings further support a diagnosis of aspiration pneumonia because the opacities
are located in the superior regions of the lower lobes, which (in addition to the
posterior regions of the upper lobes) are the most dependent locations in the lungs
of supine individuals. Anaerobic bacteria (Peptostreptococcus, Bacteroides,
Prevotella, and Fusobacterium) are the dominant organisms in the upper airway and
may be isolated from cultures in patients with anaerobic pneumonia.
(Choices A and C) Reduced intercostal muscle strength can result in atelectasis and
hypoxia. Extended immobility can produce atelectasis in the posterior lungs if the
patient remains in a supine position. These conditions can also predispose to the
development of pneumonia if pulmonary hygiene is inadequate. However, aspiration
pneumonia secondary to dysphagia is more likely given the patient's residual
hemiparesis and soft, breathy voice (indicative of vocal cord paralysis and likely
swallow muscle dysfunction).
(Choice D) Decreased enteral feeding would result in starvation and weakness, which
do not manifest as airspace opacities on chest x-ray.
(Choice E) Impaired cellular immunity usually results in recurrent viral and fungal
infections.
Educational objective:
Elderly patients with dementia or hemiparesis may also have dysphagia, which is a
risk factor for aspiration pneumonia Dependent lung consolidation is commonly seen
in aspiration pneumonia.

Non-polar, hydrophobic amino acids such as valine, alanine, isoleucine, methionine,


and phenylalanine are generally located interiorly on globular proteins, where they
are shielded from direct contact with water. The classic plasma membrane-spanning
proteins are executors for glycoprotein hormones, such as TSH, LH, and FSH. These
G-protein-coupled membrane-bound receptors for glycoprotein hormones contain three
major domains: extracellular (responsible for ligand binding), transmembrane
(consisting of hydrophobic amino acids), and intracellular (coupled with G-
proteins).
The question stem describes repeating alpha-helical segments each composed of
approximately 20 hydrophobic amino acids; this description is characteristic of the
transmembrane region(s) of a protein (Choice C)
(Choice A) The receptors for insulin. IGF-1, and several cytokines also have three
domains: extracellular (ligand binding), transmembrane (composed of hydrophobic
amino acids), and intracellular. The intracellular (cytosolic) domain of these
membrane-associated receptor proteins contains a tyrosine kinase that is activated
upon extracellular ligand binding Once activated, tyrosine kinase will
phosphorylate available tyrosine residues.
(Choice B) Cyclic AMP formed during the activation of a Gs protein can activate
protein kinase A. Protein kinase A has the unique ability to phosphorylate and
thereby activate proteins that are capable of translocating into the nucleus. Once
within the nucleus, these special proteins bind to the promoter regions of DNA and
modulate transcription. (The proteins that actually bind to DNA are not membrane
proteins.)
(Choice D) The binding of a ligand to the extracellular domain of a transmembrane
protein causes indirect activation of the adenylate cyclase system, the opening of
ion channels, direct activation of tyrosine kinase, activation of the calcium-
calmodulin system, and activation of the inositol triphosphate system. The region
of a transmembrane protein that interacts with the ligand is found in the
extracellular space and therefore could not possibly be composed mainly of
hydrophobic amino acids.
(Choice E) This choice describes the intracellular iron-containing proteins
(hemeproteins) such as hemoglobin, myoglobin, and cytochrome oxidase. Heme is a
complex of protoporphyrin IX and iron. Hemoglobin A. the most common hemoglobin in
adults, consists of two alpha and two beta globin chains held together by non-
covalent interactions. Each subunit has stretches of alpha helices and a crevice
lined by nonpolar amino acids, where heme binding occurs.
Educational Objective:
Integral membrane proteins contain transmembrane domains composed of alpha helices
with hydrophobic amino acid residues such as valine, alanine, isoleucine,
methionine, and phenylalanine.

Nuclear chromosomes contain most of the DNA found in human cells. However,
mitochondria also contain their own DNA called mitochondrial DNA (mtDNA). This DNA
exists as a small circular chromosome with a slightly different genetic code than
that of nuclear DNA, consistent with the endosymbiotic theory that mitochondria
originated as prokaryotic cells that were later engulfed by ancient eukaryotes.
Over time, most of the genes coding for mitochondrial proteins have migrated to
nuclear DNA. However, mtDNA still codes for about 14 proteins (some involved in
oxidative metabolic pathways) and the ribosomal and transfer RNA needed for
mitochondrial protein synthesis. Each mitochondrion contains 1-10 copies of
maternally derived mtDNA. As a result, diseases arising from mutations in mtDNA are
transmitted from the mother to all of her offspring. Mitochondria can be identified
on electron microscopy by their characteristic double membrane and wavy cristae
(Choice A) The rough endoplastic reticulum has a stippled appearance secondary to
the presence of numerous ribosomes bound to its membranes. These ribosomes are
involved in the synthesis of integral membrane proteins and proteins destined for
export or packaging into granules or organelles.
(Choice B) The dark region identified within the nucleus is the nucleolus, the site
of synthesis and assembly of ribosomal components. There is no lipid membrane
separating the nucleolus from the rest of the nucleus.
(Choice C) The lighter "electron-lucent" regions within the nucleus signify
euchromatin (unpackaged DNA actively being transcribed).
(Choice E) This electron-dense membrane-bound spherical structure represents an
exocrine granule containing enzymes and other proteins packaged for secretion.
Educational objective:
Mitochondrial DNA (mtDNA) is the most common non-nuclear DNA found in eukaryotic
cells. It resembles prokaryotic DNA and is maternally derived. Mutations involving
mtDNA or nuclear DNA that codes for mitochondrial proteins can cause a variety of
mitochondrial disorders, including Leigh syndrome and MELAS.

The actions of thyroid hormones are mediated by thyroid hormone receptors, which
are located actually within the nucleus. Nuclear receptors control gene expression
by binding to DNA at hormone-responsive elements in the promoter region of target
genes. Other molecules that act through nuclear receptors include retinoids,
peroxisomal proliferating activated receptors, and fatty acids.
(Choice A) Tyrosine-kinase-activating receptors are located on the cell's surface
and usually have an extracellular domain for ligand binding, or transmembrane
region, and carboxy-terminal domain containing tyrosine kinase activity. Insulin
and various growth factors, such as epidermal growth factor and transforming growth
factor-beta, work through activation of tyrosine kinase.
(Choices B and E) Several peptide hormones (glucagon, PTH. ACTH. and gonadotropins)
act via these G-protein-coupled membrane-bound receptors. These receptors typically
have seven transmembrane regions spanning the plasma membrane with one amino-
terminal domain and one carboxy-terminal domain. The extracellular amino-terminal
domain is responsible for hormone binding; when this occurs the cytoplasmic
carboxy-terminal actually activates G-protein, which in turn increases second
messengers, such as cyclic AMP by activation of adenylyl cyclase activity. G-
protein can also increase phospholipase C activity resulting in formation of other
second-messengers, like inositol 1,4,5-triphosphate, and diacylglycerol (DAG).
(Choice D) Receptors for several steroid hormones such as glucocorticoids,
mineralocorticoids, androgens, and estrogens are located primarily in the
cytoplasm.
Educational Objective:
Thyroid hormones alter gene transcription by binding to receptors situated inside
of the nucleus. Receptors for several steroid hormones such as glucocorticoids,
mineralocorticoids, androgens, and estrogens are usually initially present in
cytoplasm, although they do migrate to the nucleus once activated.

The actions of thyroid hormones are mediated by thyroid hormone receptors, which
are located actually within the nucleus. Nuclear receptors control gene expression
by binding to DNA at hormone-responsive elements in the promoter region of target
genes. Other molecules that act through nuclear receptors include retinoids,
peroxisomal proliferating activated receptors, and fatty acids.
(Choice A) Tyrosine-kinase-activating receptors are located on the cell's surface
and usually have an extracellular domain for ligand binding, or transmembrane
region, and carboxy-terminal domain containing tyrosine kinase activity. Insulin
and various growth factors, such as epidermal growth factor and transforming growth
factor-beta, work through activation of tyrosine kinase.
(Choices B and E) Several peptide hormones (glucagon, PTH. ACTH. and gonadotropins)
act via these G-protein-coupled membrane-bound receptors. These receptors typically
have seven transmembrane regions spanning the plasma membrane with one amino-
terminal domain and one carboxy-terminal domain. The extracellular amino-terminal
domain is responsible for hormone binding; when this occurs the cytoplasmic
carboxy-terminal actually activates G-protein, which in turn increases second
messengers, such as cyclic AMP by activation of adenylyl cyclase activity. G-
protein can also increase phospholipase C activity resulting in formation of other
second-messengers, like inositol 1,4,5-triphosphate, and diacylglycerol (DAG).
(Choice D) Receptors for several steroid hormones such as glucocorticoids,
mineralocorticoids, androgens, and estrogens are located primarily in the
cytoplasm.
Educational Objective:
Thyroid hormones alter gene transcription by binding to receptors situated inside
of the nucleus. Receptors for several steroid hormones such as glucocorticoids,
mineralocorticoids, androgens, and estrogens are usually initially present in
cytoplasm, although they do migrate to the nucleus once activated.

The response of the hepatocytes to the stimulus described in the question stem is
characteristic of the response of these cells to insulin. Insulin is an anabolic
hormone that promotes the synthesis of glycogen, triacylglycerides, nucleic acids,
and proteins. Insulin inhibits glycogenolysis and gluconeogenesis. Insulin acts via
a tyrosine kinase mechanism. The insulin cell surface receptor is a transmembrane
protein that also has cytosolic tyrosine kinase activity. The tyrosine kinase
causes phosphorylation of a poorly characterized class of proteins known as insulin
receptor substrates leading to activation of protein phosphatase. Protein
phosphatase dephosphorylates glycogen synthase thereby activating that protein and
promoting glycogen synthesis. Protein phosphatase also dephosphorylates fructose
1,6-bisphosphatase thereby inactivating that enzyme and inhibiting gluconeogenesis.
This is also a good example of how phosphorylation and dephosphorylation of enzymes
by second-messenger proteins can cause activation of some enzymes and inactivation
of others
(Choice B) Protein kinase A is the primary intracellular affector enzyme in the G-
protein / adenylate cyclase second messenger system. Increased levels of cAMP
stimulate protein kinase A to activate the necessary enzymes to carry out the
intracellular actions of the hormone that bound the cell and activated adenylate
cyclase in the first place.
(Choice C) Phospholipase C is active in the G-protein / Inositol triphosphate (IP3)
/ Calcium second
messenger system. Hormone binds its receptor and activates a G-protein that in turn
activates phospholipase C to degrade phospholipids into inositol triphosphate and
diacylglycerol Both diacylglycerol and the increased intracellular calcium caused
by IP3 will activate protein kinase C.
(Choice D) Janus protein kinase (JAK) is a part of the second messenger system for
peptide hormones such as some cytokines in a pathway referred to as JAK-STAT
(signal transducers and activators of transcription). JAK has tyrosine kinase
activity.
(Choice E) Lipoxygenase is an enzyme involved in arachidonic acid metabolism and is
responsible for the arm of that pathway that synthesizes leukotrienes.
Educational Objective:
Insulin is an anabolic hormone that acts via a tyrosine kinase second messenger
system to stimulate the synthesis of glycogen, proteins, fatty acids and nucleic
acids. Tyrosine kinase leads to the activation of protein phosphatase within cells,
and protein phosphatase directly modulates the activity of enzymes in the metabolic
Dathwavs reaulated bv insulin

In glycolysis, glucose is metabolized to pyruvate and lactate. Under aerobic


conditions, the dominant product in most tissues is pyruvate, which is converted to
acetyl CoAto enter the TCA cycle. When oxygen is depleted, such as in exercising
muscle, the dominant product is lactate. This is referred to as anaerobic
glycolysis. During glycolysis, glyceraldehyde-3-phosphate is converted to 1-3-
bisphosphoglycerate by the enzyme glyceraldehyde-3-phosphate dehydrogenase This
enzyme uses NAD-dependent oxidation, and in this reaction NAD+ is converted into
NADH. NAD+ is present in limited amounts in most cells, and it must be regenerated
from NADH for glycolysis to continue. Under aerobic conditions, NAD+ is converted
to NADH in the TCA cycle and NADH is then converted back to NAD+ in the electron
transport chain as the energy in NADH is utilized to synthesize ATP. In anaerobic
glycolysis, NAD+ is regenerated from NADH when pyruvate is converted to lactate by
lactate dehydrogenase. In strenuously exercising muscle, glycolysis can be
inhibited by limited regeneration of NAD+ from NADH (choice C).
(Choice A) During muscle contraction, glycogen is broken down by glycogen
phosphorylase for energy production via the glycolytic pathway. Epinephrine through
cyclic AMP causes phosphorylation of glycogen phosphorylase making it active. Non-
phosphorylation dependent activation of glycogen phosphorylase can occur during
muscle contraction by increased intracellular calcium concentrations and by APvlP
under extreme conditions.
(Choice B) FADH, is not produced in glycolysis. FADH, is produced from FAD during
the conversion of succinate to fumarate in the TCA cycle by the enzyme succinate
dehydrogenase.
(Choice D) Carnitine is the amino acid responsible for transport of fatty acids
into the mitochondria for beta-oxidation It is synthesized from lysine and
methionine. Vitamin C is essential for the synthesis of carnitine.
(Choice E) In glycolysis, pyruvate is formed from phosphoenolpyruvate by a
unidirectional enzyme called pyruvate kinase. Pyruvate is usually present in large
quantities in cells; therefore, it is not the limiting factor for glycolysis.
(Choice F) Citrate is formed from the condensation of acetyl CoA with oxaloacetate
in the first step of the TCA cycle. Citrate is a very powerful allosteric inhibitor
of phosphofructokinase-1. Increased citrate concentrations thereby decrease
glycolysis. In exercising muscles under anaerobic conditions, oxidative
phosphorylation of glucose through the citric acid cycle is not a dominant pathway
and hence excess citrate is not produced.
Educational Objective:
Under anaerobic conditions, NADH transfers protons to pyruvate to form lactate and
to regenerate NADT

The titration curve shown reveals three different pKas, each of which demonstrates
a buffering effect on the pH as more base is added. Thus, the amino acid in
question must have three titratable protons. Amino acids with three titratable
protons include histidine, arginine, lysine, aspartic acid, glutamic acid, cysteine
and tyrosine. Out of the answer choices, histidine is the only amino acid with
three titratable protons.
The infant in this question demonstrates an increased quantity of histidine in the
blood, which is caused by a rare autosomal recessive disorder known as
histidinemia. Patients with this disorder have a deficiency of histidase. the
enzyme required for the catabolism of histidine. Clinical manifestations can
include speech defects, psychomotor and generalized retardation, and emotional
disturbance. Of note, it is the most frequent inborn metabolic error in Japan.
Educational objective:
Amino acids with three titratable protons include histidine, arginine, lysine,
aspartic acid, glutamic acid, cysteine and tyrosine.

The regulation of insulin secretion from pancreatic beta cell is a complex process.
Glucose is the most important stimulator of insulin release. Glucose enters beta
cells by facilitated diffusion using glucose transporter 2 (GLUT-2). Glucose, after
entering beta cells, undergoes sequential oxidative metabolism through glycolysis,
followed by the citric acid cycle, which generates ATP. A high ATP to ADP ratio
within beta cells causes the closure of potassium (KATP) channels. This closure
occurs when ATP binds to the regulatory subunit of the KATP channel. KATP channels
are responsible for the outward movement of potassium from the beta cells. When
these channels close, beta cells depolarize, which results in the opening of
voltage-dependent calcium channels. High intracellular calcium then leads to
insulin release. Defects of the KATP channel gene can result in type two diabetes.
The defect causes excess function of this KATP channel; the channels do not close,
beta cells fail to be depolarized, and there is an inadequate insulin response to
glucose. Sulfonylureas are "antidiabetic" medications that are taken orally— they
work by directly binding to the regulatory subunits of KATP channels, causing them
to close.
(Choices B, C and D) Fructose 6-phosphate, pyruvate, and lactate are intermediate
products of the glycolytic pathway. Lactate is produced by anaerobic glycolysis.
Beta cells have a rich oxygen supply, and most of the glycolysis that occurs in
beta cells is aerobic. Thus, a very small amount of lactate is produced. Pyruvate
is converted into acetyl co-enzyme A, which enters the mitochondria for metabolism
via the citric acid cycle.
(Choices E, F and G) Citrate, malate, and fumarate are intermediates in the citric
acid cycle. During this process, energy is stored as NADH and FADH2. ATP is then
produced when NADH and FADH2 enter oxidative phosphorylation. The products and
intermediates of glycolysis and the citric acid cycle listed in these choices do
not have any direct effect on KATP channels.
Educational Objective:
ATP is the regulatory substance that stimulates KATP channel closure in insulin-
producing pancreatic beta cells.

One of the important actions of insulin is the facilitation of glucose uptake by


adipocytes and muscle cells.
This is achieved by the movement of cytoplasmic GLUT-4 to plasma membranes in
muscle and fat cells.
This and the many other actions of insulin are mediated through cell-surface
receptors. The insulin receptor (IR) is a tetrameric structure consisting of two
alpha and two beta subunits. The alpha subunits are extracellular and they provide
the binding site for insulin. The beta subunits are intracellular and contain
tyrosine kinase domains that are activated when insulin attaches to the alpha
subunits. A series of downstream signaling is then triggered, starting with the
autophosphorylation of IR. This is a prerequisite for activation of the receptor's
tyrosine kinase and also allows for interaction with other molecules, particularly
IRS-1 /2 (insulin receptor substrate 1 and 2).
TNF-alpha is a proinflammatory cytokine that induces insulin resistance through the
activation of serine kinases, which then result in phosphorylation of IRS-1 serine
residues. This inhibits IRS-1 tyrosine phosphorylation by insulin. Phosphorylation
of serine residues in the beta subunit of insulin receptors also hinders downstream
signaling, resulting in resistance to the normal actions of insulin Phosphorylation
of threonine residues has similar effects. Catecholamines, glucocorticoids, and
glucagon can also induce insulin resistance by this same mechanism, and
intracellular free fatty acids are suspected of increasing serine kinase activity
as well.
(Choice A) Cyclic AMP is inactivated by hydrolysis via an intracellular enzyme
called phosphodiesterase. An increase in activity of phosphodiesterase can dampen
the effect of various hormones by hydrolyzing cyclic AMP. However, cyclic AMP is
not involved in insulin action.
(Choice B) Some hormones act on membrane receptors coupled with a different subtype
of G-protein, G-2, which activates phospholipase C. This results in cleavage of
phospholipid-inositol leading to the formation of two important secondary
messengers: inositol 1.4,5 triphosphate and diacylglycerol (DAG). Inositol 1,4,5
triphosphate mediates its action by increasing intracellular calcium influx, and
DAG activates protein kinase C.
(Choice C) Hydroxyproline, a major component of bone and skin collagen, is formed
by the hydroxylation of proline, which requires vitamin C. A deficiency in vitamin
C causes poor wound healing (scurvy). Insulin plays no role in the hydroxylation of
proline.
(Choice E) Peroxisome proliferator-activated receptor gamma (PPAR gamma) belongs to
a family of nuclear receptors that alter gene transcription by binding to the
promoter region of the target gene. PPAR gamma alters the transcription of a number
of important genes, resulting in improved insulin sensitivity. A class of oral
anti-diabetic medications called thiazolidinediones, orTZDs, decrease blood sugar
by activation of PPAR gamma.

Non-glucose monosaccharides such as fructose, mannose and galactose enter the


glycolytic pathway after initial metabolism to intermediates of glycolysis. Glucose
entering the glycolytic pathway is metabolized to glucose-6-phosphate by hexokinase
or glucokinase and is then converted to fructose-6-phosphate by
phosphoglucoisomerase. Fructose-6-phosphate is converted to fructose-1,6-
bisphosphate by the unidirectional rate-limiting enzyme of glycolysis,
phosphofructokinase-1 (PFK-1). Glucose-6-phosphate, mannose-6-phosphate, galactose-
6-phosphate and glucose-1-phosphate (choices A, B, C and E) must be metabolized via
the rate-limiting PFK-1 step, which slows down their metabolism. Fructose-1-
phosphate, however, bypasses this step and therefore has the highest rate of
metabolism (choice D).
Fructose is phosphorylated on its first carbon by fructokinase in the liver
yielding fructose-1-phosphate (F1P). Aldolase B can utilize both fructose-1,6-
bisphosphate and F1P as substrates. Metabolism of fructose-1-phosphate by aldolase
B generates dihydroxy acetone phosphate (DHAP) and glyceraldehyde. DHAP is
converted by those phosphate isomerase to glyceraldehyde-3-phosphate (G3P) and
enters glycolysis. Glyceraldehyde can be phosphorylated to G3P by triokinase or
converted to DHAP; both of these compounds can enter the glycolytic pathway.
(Choice B) Mannose is generated from the metabolism of several polysaccharides and
glycoproteins. Mannose is phosphorylated to mannose-6-phosphate and then converted
to fructose-6-phosphate by the enzyme mannose isomerase.
(Choices C and E) Galactose is formed when lactose (a disaccharide of glucose and
galactose) is catabolized on the intestinal brush border by the enzyme lactase.
Galactose enters glycolysis by conversion to glucose-1 -phosphate. Galactose is
first phosphorylated by galactokinase to form galactose-1 -phosphate Subsequently,
transfer of UDP to galactose-1-phosphate occurs followed by epimerization to UDP-
glucose by galactose-4 epimerase. The resulting glucose-1-phosphate is converted to
glucose-6-phosphate (G6P) by phosphoglucomutase, and G6P enters glycolysis before
the rate limiting PFK-1 step.
Educational Objective:
Dietary fructose is phosphorylated in the liver to F-1-P and is rapidly metabolized
because it bypasses PFK-1, the rate-limiting enzyme of glycolysis. Other sugars
enter glycolysis before this rate-limiting step and are therefore metabolized more
slowly due to regulation of PFK-1.

This patient has diabetes mellitus and presents with hyperosmolar hyperglycemia, a
metabolic derangement often precipitated by infection (pneumonia in this case) and
characterized by dehydration, hyperglycemia, and hyperosmolarity without
significant ketoacidosis. His cataracts likely formed from oversaturation of the
polyol pathway secondary to long-term hyperglycemia.
Aldose reductase converts glucose into sorbitol during the first step in the polyol
pathway ot glucose metabolism. Sorbitol cannot readily cross cell membranes and is
therefore trapped inside the cells within which it is formed. If the enzyme
sorbitol dehydrogenase (sometimes referred to as polyol dehydrogenase) is also
present in the cell, it can convert sorbitol into fructose. This pathway, known as
the polyol pathway, is especially active in the seminal vesicles, as sperm use
fructose as their primary energy source. Other tissues, such as the retina, renal
papilla, and Schwann cells, have much less sorbitol dehydrogenase activity.
The human lens does contain significant levels of sorbitol dehydrogenase, (see
references), which allows for the production of fructose. However, this enzyme has
a significantly lower Vmax in the sorbitol-to-fructose direction than in the
reverse direction. When glucose levels are low, the limited forward activity of
this enzyme is sufficient to convert enough sorbitol into fructose to prevent
sorbitol accumulation In contrast, states of long-standing hyperglycemia lead to
the production of an excessive amount of sorbitol that is trapped in the cells.
This increases the osmotic pressure and facilitates the influx of water into the
lens cells, leading to the development of hydropic lens fibers that degenerate.
Eventually this results in lens opacification and cataract formation. In addition
to osmotic cell injury, oxidative stress resulting from the depletion of NADPH
contributes to cataract formation and other diabetic complications such as
neuropathy and retinopathy.
(Choice A) The end product of sorbitol metabolism is fructose, not glucose. Glucose
is formed in cells by the processes of glycogenolysis and gluconeogenesis.
(Choices C and D) Another function of aldose reductase is conversion of galactose
into galactitol (ie. this enzyme converts sugars into their corresponding sugar
alcohols). Galactitol production via this pathway is normally insignificant. In
galactosemia (galactose 1-phosphate uridyltransferase deficiency), an increased
amount of galactitol is produced, resulting in cataract formation.
(Choice E) Xylulose is an end product of glucuronic acid metabolism and an
intermediate in the pentose phosphate pathway.
Educational objective:
Aldose reductase converts glucose into sorbitol, which is further metabolized into
fructose by sorbitol dehydrogenase. This pathway is most active in the seminal
vesicles. The lens also contains significant levels of sorbitol dehydrogenase,
which become overwhelmed in the setting of hyperglycemia. Other tissues, such as
the retina, renal papilla, and Schwann cells, have much less sorbitol dehydrogenase
activity.

The endoplasmic reticulum (ER) is a continuous system of folded membranes that


enclose a single cisternal space. The rough ER (RER) has ribosomes attached and is
the site of synthesis of secretory, lysosomal, and integral membrane proteins. Once
synthesized, many proteins undergo post-translational modification inside the RER
and most are targeted for export to the Golgi apparatus. These proteins pass from
the RER to the Golgi apparatus within COP II coated transport vesicles. The Golgi
apparatus sorts and distributes proteins to the cell membrane, organelles, and
secretory granules.
In contrast to the RER. the smooth ER (SER) contains enzymes for steroid and
phospholipid biosynthesis.
All steroid-producing cells (eg, cells in the adrenals, gonads, and liver) contain
a well-developed SER. ACTH primarily stimulates the cells in the adrenal cortex to
produce glucocorticoids, but it also increases adrenal production of
mineralocorticoids and androgens to a lesser extent. These are all cholesterol-
derived steroid hormones, and the cells that synthesize them contain prominent
SERs. The SER is also involved in the detoxification of numerous drugs, especially
in hepatocytes. In addition, a specialized SER functions as the sarcoplasmic
reticulum in striated muscle cells.
(Choice A) Parathyroid hormone helps regulate serum calcium levels by increasing
osteoclastic bone resorption, increasing the distal tubular absorption of calcium,
and increasing 1,25-dihydroxyvitamin D conversion through upregulation of the renal
enzyme 1-alpha hydroxylase. Parathyroid hormone does not upregulate steroid or
lipid synthesis.
(Choice B) Progesterone is a steroid hormone secreted from the corpus luteum. It
prepares the endometrium for conception and helps maintain the new pregnancy. It
acts via a type 1 nuclear receptor that, on binding progesterone, translocates to
the nucleus and binds directly to DIMA to influence protein production. The ovarian
lutein cells that produce progesterone contain abundant smooth ER, but the
endometrial cells on which progesterone acts do not.
(Choice D) Aldosterone acts mainly on the distal tubules and collecting ducts of
the nephron to increase absorption of sodium and secretion of potassium, increasing
blood volume and blood pressure. It does not affect steroid or lipid synthesis.
(Choice E) Dopamine is a powerful vasoactive and inotropic molecule derived from
tyrosine. It functions via a G-protein-coupled receptor.
Educational objective:
In contrast to the rough endoplasmic reticulum (ER), the smooth ER contains enzymes
for steroid and phospholipid biosynthesis. All steroid-producing cells (eg, cells
in the adrenals, gonads, and liver) contain a well-developed smooth ER.

In contrast to the rough endoplasmic reticulum (ER), the smooth ER contains enzymes
for steroid and phospholipid biosynthesis. All steroid-producing cells (eg, cells
in the adrenals, gonads, and liver) contain well-develooed smooth ER.

GTP is synthesized by the citric acid cycle enzyme succinyl-CoA synthetase during
the conversion of succinyl CoA to succinate. In gluconeogenesis, the hydrolysis of
GTP is required for the phosphorylation and decarboxylation of oxaloacetate to
phosphoenolpyruvate by phosphoenolpyruvate carboxykinase.

G-protein is a heterotrimer consisting of alpha, beta, and gamma subunits


associated with the intracellular domains of cell membrane-associated receptors.
The alpha subunit of the inactivated G-protein is bound to GDP. Upon activation of
the receptor, the alpha subunit undergoes a conformational change and GDP is
released. Subsequent binding of GTP then allows for the dissociation of the alpha
subunit from the remainder of the G-protein complex. There are multiple subtypes of
alpha G-proteins, each with different secondary effects. A specific alpha subunit
known as Gs (present in the glucagon, TSH, and PTH receptor complexes) activates
adenylate cyclase when released from the G-protein complex. Once formed from ATP,
cyclic AMP activates a family of enzymes known as the cAMP-dependent protein
kinases, or protein kinase A.
Protein kinase A phosphorylates specific serine or threonine residues in some
enzymes, thereby leading to their activation or deactivation. Protein kinase A also
phosphorylates several proteins that bind to regulatory regions of genes on the DNA
molecule itself.
(Choice A) Cyclic GMP activates protein kinase G, which mediates a number of
intracellular effects including relaxation of smooth muscles, platelet activation,
sperm metabolism, and cell division. Cyclic GMP is inactivated by cGMP
phosphodiesterase, an enzyme specifically involved in cleaving cyclic GMP in the
corpus cavernosum of the penis and regulating penile erection. Drugs such as
sildenafil inhibit cGMP-specific phosphodiesterase and are used in the treatment of
erectile dysfunction.
(Choice B) Hormones such as insulin act through cell surface-associated receptors
that have intrinsic tyrosine kinase domains.
(Choice D) Cyclic AMP is cleaved by the enzyme cAMP phosphodiesterase to its
inactive form, 5'-AMP. Drugs that inhibit cyclic AMP phosphodiesterase lead to the
prolongation of the actions of cyclic AMP. An example of this is the use of
theophylline in bronchial asthma.
(Choice E) Growth hormone, erythropoietin, and cytokines such as interferon act
through cell membrane-associated receptors in the JAK-STAT pathway. JAK is a
specific intracellular tyrosine kinase associated with receptors that dimerize upon
ligand binding
Educational objective:
Protein kinase A is primarily responsible for the intracellular effects of the G-
protein / adenylate cyclase second messenger system. Some hormone receptors that
use this mechanism include the TSH, glucagon, PTH. and beta-adrenergic receptors.

G-protein is a heterotrimer consisting of alpha, beta, and gamma subunits


associated with the intracellular domains of cell membrane-associated receptors.
The alpha subunit of the inactivated G-protein is bound to GDP. Upon activation of
the receptor, the alpha subunit undergoes a conformational change and GDP is
released. Subsequent binding of GTP then allows for the dissociation of the alpha
subunit from the remainder of the G-protein complex. There are multiple subtypes of
alpha G-proteins, each with different secondary effects. A specific alpha subunit
known as Gs (present in the glucagon, TSH, and PTH receptor complexes) activates
adenylate cyclase when released from the G-protein complex. Once formed from ATP,
cyclic AMP activates a family of enzymes known as the cAMP-dependent protein
kinases, or protein kinase A.
Protein kinase A phosphorylates specific serine or threonine residues in some
enzymes, thereby leading to their activation or deactivation. Protein kinase A also
phosphorylates several proteins that bind to regulatory regions of genes on the DNA
molecule itself.
(Choice A) Cyclic GMP activates protein kinase G, which mediates a number of
intracellular effects including relaxation of smooth muscles, platelet activation,
sperm metabolism, and cell division. Cyclic GMP is inactivated by cGMP
phosphodiesterase, an enzyme specifically involved in cleaving cyclic GMP in the
corpus cavernosum of the penis and regulating penile erection. Drugs such as
sildenafil inhibit cGMP-specific phosphodiesterase and are used in the treatment of
erectile dysfunction.
(Choice B) Hormones such as insulin act through cell surface-associated receptors
that have intrinsic tyrosine kinase domains.
(Choice D) Cyclic AMP is cleaved by the enzyme cAMP phosphodiesterase to its
inactive form, 5'-AMP. Drugs that inhibit cyclic AMP phosphodiesterase lead to the
prolongation of the actions of cyclic AMP. An example of this is the use of
theophylline in bronchial asthma.
(Choice E) Growth hormone, erythropoietin, and cytokines such as interferon act
through cell membrane-associated receptors in the JAK-STAT pathway. JAK is a
specific intracellular tyrosine kinase associated with receptors that dimerize upon
ligand binding
Educational objective:
Protein kinase A is primarily responsible for the intracellular effects of the G-
protein / adenylate cyclase second messenger system. Some hormone receptors that
use this mechanism include the TSH, glucagon, PTH. and beta-adrenergic receptors.

In the inactivated state, cortisol receptors lie within the cytoplasm in


association with heat shock proteins. When cortisol binds its carboxy terminal to
the receptor, the heat shock proteins are liberated, the receptors homodimerize.
and the dimers then translocate to within the nucleus. These receptors can now
control gene expression by binding to DNA at the hormone-responsive elements in the
promoter region of target genes. Cortisol increases the transcription of enzymes
involved in gluconeogenesis (formation of glucose from fat and protein substrate).
(Choice A) Insulin acts on a cell surface receptor with resultant activation of
tyrosine kinase. One of the principal effects of insulin is suppression of
gluconeogenesis.
(Choice C) Growth hormone receptors are also membrane-bound. Attachment of growth
hormone to the extra cellular domain of the growth hormone receptor results in
receptor dimerization and conformational change in the Janus kinase (JAK), which
stimulates the tyrosine kinase activity and activates down-stream pathways
including STAT transcription activity, and others. Growth hormone increases
gluconeogenesis by increasing lipolysis (gluconeogenesis substrate) and
gluconeogenesis in the liver
(Choices D, E and F) Catecholamines (epinephrine and norepinephrine) and glucagon
act via G-protein-coupled, membrane-bound receptors. Hormone binding to the surface
domain results in activation of G-protein, which in turn, increases second
messengers, such as cyclic AMP by activation of adenylyl cyclase activity. G-
protein can also increase phospholipase C activity resulting in formation of two
other second-messengers: inositol 1,4,5-triphosphate and diacylglycerol (DAG).
Catecholamines, particularly epinephrine and glucagon, increase glycogenolysis
(breakdown of glycogen). These hormones are the major counter-regulatory hormones
for protection against hypoglycemia. These hormones are secreted in response to
hypoglycemia, and increase blood glucose predominantly by increasing
glycogenolysis.
Educational Objective:
Receptors for cortisol are located within the cytoplasm and are translocated to the
nucleus after binding to cortisol. In the nucleus, the cortisol-receptor complex
binds to the hormone responsive elements, causing an alteration in the
transcription of target genes.

Biotin acts as a C02 carrier on the surface of the carboxylase enzyme, which
encompasses the enzymatic subtypes acetyl-CoA carboxylase (ACC), pyruvate
carboxylase (PC), propionyl carboxylase (PCC), and beta-methylcrotonyl CoA
carboxylase (MCC). All of these enzymes play roles in carbohydrate and lipid
metabolism. In the tissues responsible for gluconeogenesis, for instance, pyruvate
carboxylase (and therefore biotin) is necessary for the conversion of pyruvate to
oxaloacetate. In biotin-deficient individuals, the level of pyruvate rises and the
pyruvate is converted to lactic acid instead. Metabolic acidosis results.
Another example of metabolic derangement secondary to biotin deficiency is the need
for propionyl CoA carboxylase (and therefore biotin) to synthesize succinyl CoA
from amino acids such as valine. In biotin-deficient individuals, the propionyl CoA
builds up and is instead metabolized into a surplus of odd-chain fatty acids.
Deficiencies in this cofactor are rare, but can occur secondary to poor diet,
excessive raw egg white consumption (due to the high levels of biotin-binding
avidin in egg whites), and congenital disorders of biotin metabolism.
(Choices A, C, D, and E) These conversions are not impaired in patients with biotin
deficiency.
Educational Objective:
Biotin acts as a C02 carrier on the surface of the carboxylase enzyme and is
necessary for numerous conversions, including pyruvate to oxaloacetate. Excessive
ingestion of avidin (which is found in egg whites) has been associated with biotin
deficiency.

Pyruvate dehydrogenase (PDH) is an allosteric enzyme that converts pyruvate into


acetyl-CoA in the presence of oxygen (i.e. during aerobic metabolism). In the
absence of oxygen or with a deficiency of PDH, pyruvate is alternatively converted
to lactate by the enzyme lactate dehydrogenase. Excessive lactate production in
these states results in lactic acidosis.
PDH deficiency has a wide spectrum of presentations. Because carbohydrates may
aggravate lactic acidosis, a ketogenic diet is recommended in these patients. Amino
acid catabolism following removal of the amino group results in formation of
intermediates that are either glucogenic (producing intermediates of the citric
acid cycle or pyruvate) or ketogenic (producing acetoacetate or its precursors).
Some amino acids such as phenylalanine, isoleucine and tryptophan are both
glucogenic as well as ketogenic Leucine and lysine (Choice D) are exclusively
ketogenic and would not lead to increased formation of lactic acid. Lysine is an
essential amino acid that is totally ketogenic.
(Choice A) Glycerol forms the backbone to triglycerides, and in the fasting state
when triglycerides are degraded for energy, glycerol is released to be used in
gluconeogenesis. Glycerol can aggravate lactic acidosis in patients with pyruvate
dehydrogenase deficiency because it can be converted to dihydroxyacetone phosphate,
which in turn can form pyruvate and subsequently lactate.
(Choices B and F) Alanine is a non-essential glucogenic amino acid that can be
transaminated to form pyruvate. Similarly, serine is a non-essential amino acid
that can be converted to pyruvate by serine hydratase.
(Choice C) Galactose, when combined with glucose as a disaccharide, forms the milk
sugar lactose. Galactose is metabolized in the glycolytic pathway, thus galactose
feeding will result in aggravation of lactic acidosis in patients with pyruvate
dehydrogenase deficiency.
(Choice E) Asparagine is a non-essential glucogenic amino acid that is catabolized
initially to aspartate by the enzyme asparaginase. Aspartate is transaminated with
alpha ketoglutarate to produce glutamate and oxaloacetate, a gluconeogenic
intermediate and an intermediate in the TCA cycle.
Educational Objective:
Pyruvate dehydrogenase deficiency is a disease with multiple possible presentations
ranging from neonatal death to mild episodic symptoms in adulthood. By preventing
the conversion of pyruvate to acetyl CoA, pyruvate is shunted to lactic acid
resulting in lactic acidosis in these patients. Lysine and leucine are exclusively
ketogenic and would not increase the blood lactate level in patients suffering from
pyruvate dehydrogenase deficiency.

This electron micrograph displays details of pancreatic exocrine cell intracellular


organelles. Four nuclei are visible and within two of the four nuclei, there is a
dense round structure, the nucleolus (B) The nucleolus is not separated from the
nucleoplasm by a membrane and is composed of both ribosomal DNA and ribosomal
proteins. The primary function of the nucleolus is the synthesis of ribosomal RNA.
Other items identified in this micrograph include the rough endoplasmic reticulum
(A), the nucleus (C), mitochondria (D), and a zymogen granule (E).
The characteristics of the nucleus that make it identifiable are the presence of a
nucleolus. The electron lucent portion of the nucleolus is composed of euchromatin,
while the electron dense regions are composed of heterochromatin. Heterochromatin
refers to DNA that is not being actively transcribed; euchromatin is DNA that is
being actively transcribed, with the mRNA subsequently translated into proteins in
the cytoplasm and on the rough ER
The rough endoplasmic reticulum (RER) is identifiable because of its characteristic
folded membranes whose surface is coated with ribosomes, giving it a "speckled" or
"rough" appearance. Proteins synthesized on the RER are generally destined for
export to the extracellular space or to lysosomes.
Mitochondria are organelles with dual phospholipid bilayer membranes that can be
recognized by the presence of foldings in their inner membranes, called cristae.
Educational Objective:
The nucleolus, the dark intranuclear body visible both by light microscopy and
electron microscopy, is the site of ribosomal RNA synthesis.

Glucose is the major stimulant of insulin secretion. After glucose enters the beta
cells, it is metabolized by glycolysis, followed by citric acid cycle, which
results in the generation of ATP molecules. A high ATP to ADP ratios within the
beta cells results in the closure of potassium (KATP) channel, which causes insulin
secretion by opening the voltage-dependent calcium channel.
In the beta cell of the pancreas, the first enzyme in the glycolytic pathway is
glucokinase, instead of hexokinase, the first enzyme of other tissues. Glucokinase
converts glucose to glucose 6-phosphate. Glucokinase has a much higher Km (10 mmol)
than hexokinase (0.5 mmol). Furthermore, glucokinase is less sensitive to
allosteric inhibition by its product, glucose 6-phosphate. Glucokinase serves as a
major glucose-sensor for the beta cell. An increase in serum glucose leads to
increased metabolism of glucose within the beta cells, ultimately leading to
insulin secretion by the formation of ATP. The conversion of glucose to glucose-6-
phosphate by glucokinase is crucial and rate limiting. Once metabolized to glucose
6-phosphate, glucose is rapidly metabolized in the glycolytic and citric cycle.
Glucokinase defects result in one of the types of maturity-onset diabetes.
Inactivating mutations of the glucokinase gene lead to a decrease in the enyzme's
function, causing decrease in the metabolism of glucose, lesser ATP formation, and
diminished insulin secretion.
(Choices A - G) The other choices listed are enzymes involved in glycolytic and
gluconeogenic pathways. Diabetes mellitus is not a complication of defects in these
enzymes.
Educational Objective:
Glucokinase is a glucose sensor within pancreatic beta cells. Inactivating
mutations of the enzyme result in mild hyperglycemia that can be exacerbated by
pregnancy.

The G-protein-coupled-receptors have a very characteristic structure with seven


transmembrane regions, an extracellular domain and an intracellular domain coupled
with the trimeric G-protein. In their inactivated state, G-proteins exist as
heterotrimers consisting of alpha, beta and gamma subunits with guanosine
diphosphate (GDP) tightly bound to the alpha subunit G proteins are activated after
binding of hormone to the extracellular domain. The first step in activation of a
G-protein occurs when GDP is exchanged for GTP on the alpha subunit. Once bound to
GTP, the alpha subunit dissociates from the beta and gamma subunits and exposes its
catalytic domain for either adenylate cyclase or phospholipase C depending on the
ligand.
If the G-protein alpha subunit activates phospholipase C, then the degradation of
phosphatidylinositol 4,5-bisphosphate to inositol 1,4,5-triphosphate (IP3) and
diacylglycerol (DAG) occurs. Diacylglycerol stimulates protein kinase C, which is
responsible for some intracellular effects. Inositol 1,4,5-triphosphate (IP;)
produces most of the intracellular effects of this pathway by increasing
intracellular calcium, and elevated intracellular calcium activates protein kinase
C. If the action of IP3 were blocked as described in the question stem, then
decreased activation of protein kinase C would occur upon hormone binding. (Choice
C).
(Choice A) The activity of phospholipase C would be unchanged if IP3 were blocked
because phospholipase C exerts its effect before IP. in the calcium /
phosphatidylinositol second messenger system.
(Choice B) Lipoxygenase is an enzyme responsible for formation of leukotrienes from
arachidonic acid. It is not involved in intracellular signaling.
(Choice D) Termination of the effects of hormones that act by cAMP or cGMP G-
protein second messenger systems is carried out by the enzyme phosphodiesterase.
Phosphodiesterase has no effect on the IP3 second messenger system.
(Choice E) Activation of adenylate cyclase leads to the formation of cyclic AMP and
the subsequent activation of protein kinase A. Protein kinase A activates the
proteins that produce the intracellular effects of hormones.
Educational Objective:
After a hormone binds a G-protein coupled receptor that activates phospholipase C,
the initial step of the IP5 second messenger system involves degradation of
membrane lipids into diacylglycerol (DAG) and inositol triphosphate (IP3) by that
enzyme. Protein kinase C is activated by DAG as well as calcium released from
sarcoplasmic reticulum under the influence of IPy

Fructose is obtained in the diet mainly from fruits and table sugar (sucrose).
Fructose is absorbed in the proximal intestine by the facilitative hexose
transporter GLUT 5. It is phosphorylated by fructokinase in the liver yielding
fructose-1-phosphate (F1P), and metabolism of fructose-1-phosphate by aldolase B
generates dihydroxyacetone phosphate (DHAP) and glyceraldehyde. Glyceraldehyde and
DHAP can be converted to glyceraldehyde-3-phosphate (G3P), which can then be
metabolized in the glycolytic pathway.
Fructokinase deficiency causes essential fructosuria, a rare autosomal recessive
asymptomatic disorder. Fructose from the diet is absorbed in the gut and excreted
unchanged in the urine. In normal liver, a small amount of fructose is
phosphorylated to fructose-6-phosphate, which is a glycolytic intermediate. With
fructokinase deficiency, the metabolism of fructose to fructose-6-phosphate (F6P)
by the enzyme hexokinase is the principal method of metabolism of dietary fructose.
F6P can be metabolized in the glycolytic pathway or converted to glucose-6-
phosphate (G6P) by phosphoglucoisomerase. G6P can be metabolized in the following
pathways: 1) Conversion to G1P by phosphoglucomutase (G1P can be used for glycogen
synthesis), 2) Conversion to glucose by glucose-6-phosphatase (this enzyme is
absent in muscles), 3) Conversion to 6-phosphogluconolactone by glucose-6-phosphate
dehydrogenase in the first step of the pentose phosphate pathway.
(Choice C) Aldolase B can use F1P and F1, 6 BP as substrates; it is not directly
involved in glycogen synthetic pathway.
(Choice D) UDP-galactose-4 epimerase is involved in the metabolism of galactose.
Galactose is phosphorylated by galactokinase to form galactose-1-phosphate, which
is then converted to glucose-1-phosphate (G1P) by galactose-1-phosphate uridyl
transferase. This reaction generates UDP-galactose and G1P. UDP-galactose-4-
epimerase converts UDP-galactoseto UDP-glucose.
(Choice E) Aldose reductase is the enzyme that converts glucose to sorbitol. This
enzyme plays a role in the development of chronic complications of diabetes. Aldose
reductase has a low affinity for glucose, and in nondiabetic individuals very small
amounts of glucose are metabolized by this enzyme. In diabetics, the amount of
glucose metabolized by the aldose reductase pathway increases significantly due to
chronically high blood glucose concentrations.
Educational Objective:
In patients with essential fructosuria, metabolism of fructose by hexokinase to
fructose-6-phosphate is the primary method of metabolizing dietary fructose; this
pathway is not significant in normal individuals.
'
Receptors are subdivided into four major categories: steroid, ion channel, enzyme-
linked, and G-protein-linked. Steroid receptors are located in the cell cytoplasm
or nucleus, while the other three receptor types are found on the cell surface.
Enzyme-linked receptors are proteins that span the cell membrane, extruding an
extracellular terminal that binds to the corresponding growth factor. Once bound,
the receptor protein configuration is changed, which triggers a cascade of events.
Some of the enzyme-linked receptors are enzymes themselves, and some activate
enzymes present in the cytosol. The differences between these two types of enzyme-
linked receptors are as follows:
Receptor With intrinsic enzyme activity (receptor tyrosine kinase) Without
intrinsic enzyme activity (tyrosine-kinase associated receptor)
Structure Extracellular domain (binds the growth factor) Extracellular domain
Transmembrane domain Transmembrane domain
Cytosolic domain (enzyme) Cytosolic domain (lacks enzymatic activity)
Signaling pathway MAP-kinase Receptor autophosphorylates and triggers
phosphorylation of Ras protein JAK/STAT Receptor activates Janus kinases
(JAKs), which phosphorylate STATs (signal transducers and activators of
transcription)
Examples Growth factor receptors: EGF, PDGF, FGF, etc. Receptors for cytokines,
growth hormone, prolactin. IL-2
(Choices A and B) The insulin receptor and platelet-derived growth factor receptor
have extracellular ligand-binding domains, transmembrane segments, and cytoplasmic
tails that demonstrate intrinsic enzyme activity. The JAK/STAT signal transduction
system is not used by these receptors.
(Choice D) The atrial natriuretic peptide receptor has intrinsic guanylate cyclase
activity. The JAK/STAT signal transduction system is not used by this receptor.
(Choice E) The progesterone receptor requires the ligands to diffuse through the
cell membrane before binding can occur. Transcription is then activated. The
JAK/STAT signal transduction system is not used by this receptor.
(Choice F) The gamma-aminobutyric acid (GABA) receptor uses either a ligand-gated
ion channel or G protein coupling. The JAK/STAT signal transduction system is not
used by this receptor.
Educational Objective:
Colony-stimulating factors, prolactin, growth hormones and cytokines utilize
tyrosine kinase-associated receptors and the JAK/STAT signaling pathway.

The vascular endothelium plays an important role in vasodilation mediated by


acetylcholine, bradykinin, serotonin, substance P, and shear forces. These stimuli
activate specific membrane receptors present on endothelial cells and trigger
cytosolic calcium influx. This causes the activation of endothelial nitric oxide
synthase (eNOS), which synthesizes nitric oxide from arginine, NADPH, and 02.
Nitric oxide then diffuses to smooth muscle cells, where it activates soluble
guanylyl cyclase and increases formation of cyclic GMP. High levels of cyclic GMP
activate protein kinase G, reducing cytosolic calcium levels and causing relaxation
of vascular smooth muscle cells.
The availability of L-argininefor synthesizing nitric oxide depends on several
factors, including exogenous food intake, endogenous synthesis, intracellular
storage and degradation, and the presence of asymmetrical dimethyl arginine (an
endogenous analog of L-arginine. which works as a competitive inhibitor of eNOS).
Important amino acid derivatives:
m Arginine —*■ Nitric oxide
• Arginine + Aspartate —»Urea
• Glycine + Succinyl CoA —> Heme
• Glycine + Arginine + SAM —»■ Creatine m Glutamate —► GABA
9 Glutamate —» Glutathione
• Glutamate + Aspartate —* Pyrimidines
• Glutamate + Aspartate + Glycine —*■ Purines
• Histidine —* Histamine
m Phenylalanine —»Tyrosine —>■ DOPA —* Dopamine
• Tryptophan —► Serotonin —> Melatonin
• Tryptophan -+ Niacin
9 Tyrosine —> Thyroxine, Melanin
Educational objective:
Nitric oxide is synthesized from arginine by nitric oxide synthase. As a precursor
of nitric oxide, arginine supplementation may play an adjunct role in the treatment
of conditions that improve with vasodilation, such as stable angina.

Thiamine deficiency is associated with infantile and adult beriberi, as well as


Wernicke-Korsakoff syndrome in alcoholics. Manifestations of infantile beriberi
appear between the ages of two and three months and include a fulminant cardiac
syndrome with cardiomegaly, tachycardia, cyanosis, dyspnea, and vomiting.
Adult beriberi is categorized as dry or wet. Dry beriberi describes a symmetrical
peripheral neuropathy accompanied by sensory and motor impairments, especially of
the distal extremities. Wet beriberi includes this neuropathy as well as cardiac
involvement (eg, cardiomegaly, cardiomyopathy, congestive heart failure, peripheral
edema, tachycardia).
(Choice A) Vitamin A deficiency is characterized by night blindness, xerophthalmia,
and vulnerability to infection (especially measles).
(Choice C) Vitamin B, (riboflavin) deficiency is characterized by cheilosis,
stomatitis, glossitis, dermatitis, corneal vascularization, and ariboflavinosis.
(Choice D) Pyridoxine (vitamin B6) deficiency is characterized by cheilosis,
glossitis, dermatitis, and peripheral neuropathy.
(Choice E) Niacin deficiency is characterized by pellagra (dementia, dermatitis,
and diarrhea).
(Choice F) Ascorbic acid (vitamin C) deficiency is characterized by scurvy
(hemorrhages, bleeding into joint spaces, gingival swelling, impaired wound
healing, and weakened immune response to local infections).
(Choice G) Vitamin B12 (cobalamin) deficiency is frequently associated with
pernicious anemia. The classic presentation of pernicious anemia is an older,
mentally slow woman of northern European descent who is "lemon colored” (anemic and
icteric), has a smooth, shiny tongue indicative of atrophic glossitis, and
demonstrates a shuffling broad-based gait.
Educational Objective:
High-output congestive heart failure and neurological symptoms are strongly
suggestive of wet beriberi (thiamine deficiency).

Methionine is a unique amino acid that can be degraded to succinyl-CoA (an


intermediate of the citric acid cycle) and S-adenosyl-methionine (SAM) (a major
methyl donor in single carbon metabolism). SAM has an activated methyl group that
can be transferred to a variety of acceptor molecules. After the transfer of a
methyl group, S-adenosyl-methionine is converted into S-adenosyl-homocystine, and
S-adenosyl-homocystine is then broken down to form adenosine and homocysteine.
Subsequently, the conversion of homocysteine to cystathionine requires an enzyme
called cystathionine synthetase, the amino acid serine, and the presence of the
cofactor Vitamin B6. Cystathionine is then converted to cysteine by the enzyme
cystathionase. which also requires Vitamin B6 as a cofactor. Alternatively, using
vitamin B12 as a cofactor, homocysteine can combine with N-5-methyl-
tetrahydrofolate to form tetrahydrofolate, a substance needed for purine and
thymine synthesis.
The patient described in this vignette most likely has homocystinuria, a condition
which leads to hypercoagulability and premature atherosclerosis. The most common
cause of homocystinuria is a defect in cystathionine beta synthetase, the enzyme
that converts homocystine to cystathionine. Individuals who have a complete
deficiency of cystathionine beta synthetase exhibit premature atherosclerotic
disease, ectopia lentis (displacement of the ocular lens), osteoporosis, and mental
retardation. These patients cannot form cysteine from homocysteine; thus, cysteine
is essential in the diet of these patients (Choice A).
(Choices B and E) Glycine is one of the nonessential glucogenic amino acids that
can be converted into serine by the addition of methylene group from methylene-
tetrahydrofolic acid. Serine is also a nonessential glucogenic amino acid which in
combination with homocystine leads to the formation of cystathionine and L-cystine.
Glycine and serine metabolism are not deranged in patients with homocystinuria.
(Choice C) Glutamate is a glucogenic nonessential amino acid that can be converted
into proline by cyclization and reduction reactions. Glutamate is the primary
acceptor of amino groups during the degradation of other amino acids.
(Choice D) Hydroxylation of the essential amino acid phenylalanine leads to the
formation of tyrosine in a reaction catalyzed by phenylalanine hydroxylase.
Remember that deficiency of phenylalanine hydroxylase is the most common cause of
phenylketonuria (PKU).
(Choice F) Asparagine is an amino acid formed from aspartate by the enzyme
asparagine synthetase, using glutamine as an amino group donor. Asparagine becomes
essential for rapidly dividing tumor cells that can not produce the amino acid
rapidly enough on their own. This fact is exploited with the drug asparaginase,
which decreases asparagine concentration in tumor cells and leads to subsequent
lysis of these rapidly arowina cells.

Familial dysbetalipoproteinemia (type III hyperlipoproteinemia) is characterized


clinically by xanthomas and premature coronary and peripheral vascular disease. The
primary defects in familial dysbetalipoproteinemia are in ApoE3 and ApoE4,
apolipoproteins found on chylomicrons and VLDL that are responsible for binding
hepatic apolipoprotein receptors. Without ApoE3 and ApoE4, the liver cannot
efficiently remove chylomicrons and VLDL remnants from the circulation, causing
their accumulation in the serum and resultant elevations in cholesterol and
triglyceride levels.
(Choice A) Chylomicrons, particles composed primarily of triacylglycerol, are
synthesized on the RER and Golgi apparatus of small intestinal enterocytes. They
are released from enterocytes with only ApoB-48 apolipoprotein; they subsequently
receive ApoC-ll and ApoE from HDL particles.
(Choice C) ApoB-100 is present on LDL and is required for receptor-mediated uptake
of LDL by extrahepatic tissues.
(Choice D) Lipoprotein lipase is activated by ApoC-ll carried by chylomicrons and
VLDL. ApoC-ll deficiency results in hyperchylomicronemia (type 1
hyperlipoproteinemia).
(Choice E) ApoA-l is required for esterification of free cholesterol in HDL
particles by lecithin-cholesterol acyltransferase (LCAT). ApoA-l and LCAT
deficiencies result in low HDL and increased circulating free cholesterol levels.
Educational objective:
The key functions of important apolipoproteins are as follows:
ApoA-l: LCAT activation (cholesterol esterification)
ApoB-48: Chylomicron assembly and secretion by the intestine ApoB-100: LDL particle
uptake by extrahepatic cells ApoC-ll: Lipoprotein lipase activation
ApoE-3 & -4: VLDL and chylomicron remnant uptake by liver cells

Methionine and tetrahydrofolate are formed when methyl-tetrahydrofolate donates a


methyl group to homocysteine. Tetrahydrofolate rapidly accepts one-carbon moieties
to re-enter the cycle. The conversion of homocysteine to methionine requires the
cofactor vitamin B12 (cobalamin). In vitamin B12 deficiency, tetrahydrofolate
cannot be regenerated. Folate metabolism is consequently impaired.
Defective DNA synthesis and its resultant megaloblastic erythropoiesis is seen in
both vitamin B12 and folate deficiency. Homocysteine levels are elevated in both
conditions as well. Elevated homocysteine is a risk factor for arterial and venous
thrombosis.
Vitamin B12 is alone responsible for the conversion of methylmalonyl CoAto succinyl
CoA. Thus, in vitamin B12 deficiency, methylmalonyl CoA levels are elevated. The
result is the incorporation of nonphysiologic fatty acids into neuronal lipids.
This contributes to the neurologic dysfunction in vitamin B12 deficiency as well.
Importantly, whereas homocysteine is elevated in both folic acid and vitamin B12
deficiency, methylmalonyl CoA is elevated in vitamin B12 deficiency only.
Educational Objective:
Homocysteine is converted to methionine using methylcobalamin and methyl
tetrahydrofolate.

At least three specific small bowel enterocyte apical transport proteins appear to
be involved in the absorption of amino acids from the diet. In Hartnup disease, the
intestinal and renal absorption of tryptophan is defective. Tryptophan is an
essential amino acid and a precursor for nicotinic acid, serotonin, and melatonin.
The clinical manifestations of Hartnup disease are primarily due to the
malabsorption of tryptophan, resulting in niacin (Vitamin B3) deficiency, because
niacin is synthesized from tryptophan.
Most children with Hartnup disease are asymptomatic, but some children experience
photosensitivity and pellagra-like skin rashes as in the case described above
Neurologic involvement can occur most commonly leading to ataxia Neurologic and
skin symptoms typically wax and wane during the course of this disease. The main
laboratory finding in Hartnup disease is aminoaciduria, restricted to the neutral
amino acids (alanine, serine, threonine, valine, leucine, isoleucine,
phenylalanine, tyrosine, tryptophan, and histidine). The urinary excretion of
proline, hydroxyproline, and arginine remains unchanged, and this important finding
differentiates Hartnup disease from other causes of generalized aminoaciduria such
as Fanconi syndrome. Treatment with nicotinic acid or nicotinamide and a high-
protein diet generally results in significant improvement of symptoms.

Nucleosomes are structural subunits present inside the nucleus composed of nuclear
proteins called histones. There are five major subtypes of histones: H1, H2A, H2B,
H3, and H4. The nucleosome core is composed of two molecules each of H2A, H2B, H3,
and H4, making eight total histone proteins in each nucleosome core. During the
initial steps of DNA packaging into chromatin, the DNA double helix wraps around
the nucleosome core twice, but in contrast to the other histone proteins, H1
histones are not part of the nucleosome H1 histones participate in DNA packaging by
binding the segment of DNA that lies between nucleosomes and facilitating the
packaging of nucleosomes into more compact structures. The association of DNA with
histones gives the appearance of a "beaded chain," as this structure undergoes
further rounds of coiling and association with other structural proteins, such as
nuclear scaffold proteins, before ultimately forming chromosomes.
(Choice A) Chromatin also contains non-histone proteins, such as enzymes that are
required for DNA replication and transcription. Topoisomerase II, also called DNA
gyrase, reduces DNA strand tension during DNA replication.
(Choice B) snRNPs, also known as "snurps," are small nuclear ribonucleoproteins
that bind pre-mRNA and participate in the formation of spliceosomes, which
participate in the processing of pre-mRNA into mature RNA.
(Choice C) Ubiquitin is a small protein present in the cytoplasm and nucleus of all
eukaryotes that attaches covalently to various proteins and provides intracellular
signals for the programmed degradation of 'tagged' proteins by the proteasome.
Educational Objective:
Histone H1 is located outside of the nucleosome core and helps to package
nucleosomes into more compact structures by binding and linking DNA between
adjacent nucleosomes.

Telomerase is a reverse transcriptase enzyme (RNA-dependent DNA polymerase) that


adds TTAGGG repeats to the 3' end of DNA strands at the terminal end of
chromosomes, the telomere region. Telomerase is similar to other reverse
transcriptase enzymes in that it synthesizes single-stranded DNA using single-
stranded RNA as a template Telomerase is composed of two main subunits: the reverse
transcriptase TERT and the RNA template TERC. The TERC template is a "built-in"
part of the telomerase enzyme. The TERC RNA template is repeatedly read by the
reverse transcriptase to add TTAGGG DNA sequence repeats to the ends of chromosomes
(telomeres).
Stem cells have very long telomeres and active telomerase. but with every cell
division, the length of telomeres progressively shortens. Terminally differentiated
adult somatic cells have very short telomeres. Critical shortening in telomere
length is thought to be one signal for programmed cell death. On the other hand,
cancer cells up-regulate their telomerase activity, preventing cellular death by
maintaining the length of their telomeres. Cancer cells are considered immortal
because these cells continue to divide without aging or shortening of their
telomeres, thus, telomerase is a potential target for the treatment of cancers
Syndromes of premature aging such as Bloom syndrome are associated with shortened
telomeres.
Stem cells are undifferentiated cells that have the potential to differentiate into
other cell types. Embryonic and adult stem cells are two major types of stem cells.
While embryonic stem cells are present in the very early stages of embryogenesis
and give rise to every cell type in adult humans, adult stem cells are thought to
be present in most tissues where they are responsible for replacement of dead
cells. For instance, the epidermis is continuously replaced from stem cells present
in the basal cell layers. Similarly, bone marrow stem cells replace peripheral red
and white blood cells. Stem cells have very long telomeres; they are able to
proliferate indefinitely in a controlled manner, giving rise to replacement stem
cells and daughter cells that differentiate into the desired tissue.
(Choices B, C and E) Pancreatic beta-cells, neurons, and myocardial cells have a
very low potential to replicate and have low telomerase activity.
(Choice D) Erythrocytes are devoid of a nucleus and have no potential to divide.
Educational Objective:
Telomerase is an enzyme that possesses reverse transcriptase (RNA-dependent DNA
polymerase) activity and is normally expressed in stem cells as well as cancer
cells. However, cancer cells are immortal because these cells continue to divide
without aging and shortening of their telomeres.

Changes in the genetic code can result in the formation of altered proteins. For
instance, the protein formed in the cell culture described in the question stem is
a larger, nonfunctional protein. The normal protein is shorter and functional. This
is explained by mutations at the splice site (Choice E). After transcription, mRNA
contains sequences from both introns and exons; this type of RNA is called pre-RNA
or heteronuclear RNA (hnRNA). The pre-RIMA must be processed to mature mRNA by
posttranscriptional modifications including 5' methylguanosine capping, addition of
a 3' polyadenine (Poly A) tail, and splicing Only exons contain the proper base
pairs in the correct order that will result in the formation of an appropriate
functional protein; therefore, introns are excised before translation by a process
known as splicing. Mutation of splice sites result in the formation of larger
proteins that are usually nonfunctional, but often retain the immunoreactivity of
the normal protein (binding to antibodies).
(Choice A) Silent mutations result in no changes in formed proteins. The proteins
are functional and have the same size.
(Choice B) Missense mutations are characterized by a change in the code through
base substitution, resulting in an amino acid change. For instance, changing UUU to
UCU changes the translated amino acid from phenylalanine (UUU) to serine (UCU). The
translated proteins may be dysfunctional but usually retain the same size.
(Choice C) Frameshift mutations occur with a deletion, or less commonly, an
insertion of base pairs which are not a multiple of three. Frameshift mutations
alter the reading frame of the genetic code usually resulting in the formation of
shorter, nonfunctional proteins.
(Choice D) Nonsense mutations introduce a stop codon within a gene sequence,
resulting in the formation of shorter, nonfunctional proteins.
Educational Objective:
Splice site mutations frequently result in the production of larger proteins with
altered function but preserved immune reactivity.

This patient most likely has porphyria cutanea tarda, the most common disorder of
porphyrin synthesis. Enzyme deficiencies in the early steps in porphyrin synthesis
cause abdominal pain and neuropsychiatric manifestations without photosensitivity,
while late step derangements (after the condensation of porphobilinogen) cause
photosensitivity. More specifically, defects in URO decarboxylase, COPRO oxidase,
PROTO oxidase, and Ferrochelatase result in photosensitivity. Photosensitivity
induced by porphyria is thought to be mediated by the formation of porphyrin-
mediated superoxide free radicals from oxygen upon exposure to sunlight.
(Choice A) Deficiency of ALA-synthase will result in a decrease in formation of all
porphyrins. This deficiency will not result in porphyria but will result in a
decrease in heme synthesis and concurrent hypochromic, microcytic anemia. Pyridoxal
phosphate (Vitamin B6) is the cofactor required for activity of ALA synthase; thus,
pyridoxine deficiency can result in microcytic hypochromic anemia secondary to
decreased heme synthesis.
(Choices B and C) Deficiencies in ALA dehydrase and HMB synthase do not result in
photosensitivity because the metabolites that accumulate in these enzyme
deficiencies are not porphyrinogens or porphyrins and are therefore unable to react
with oxygen upon excitation by ultraviolet light.
(Choice E) Bilirubin glucuronyltransferase is a hepatic enzyme responsible for the
conjugation of bilirubin with glucuronide, improving solubility for biliary
excretion. A decrease in glucuronyltransferase results in unconjugated
hyperbilirubinemia.
Educational Objective:
Enzyme deficiencies of the early steps in porphyrin synthesis cause
neuropsychiatric manifestations without photosensitivity, while late step
derangements lead to photosensitivity. Photosensitivity in porphyria causes vesicle
and blister formation on sun-exposed areas as well as edema, pruritus, pain and
erythema.

After transcription, the preliminary, unprocessed mRNA is known as precursor mRNA,


or heterogeneous nuclear RNA (hnRNA). Eukaryotic pre-mRNA undergoes significant
posttranscriptional processing before leaving the nucleus, including 5'-capping,
poly A tail addition, and intron splicing.
Once mRNA is finalized, it leaves the nucleus bound to specific packaging proteins.
Upon entering the cytoplasm, these mRNA complexes often associate with ribosomes to
undergo translation. However, certain mRNA sequences instead associate with
proteins that are found in P bodies. P bodies are distinct foci found within
eukaryotic cells that are involved in mRNA regulation and turnover They play a
fundamental role in translation repression and mRNA decay, and contain numerous
proteins including RNA exonucleases, mRNA decapping enzymes, and constituents
involved in mRNA quality control and microRNA-induced mRNA silencing. P bodies also
seem to function as a form of mRNA storage, as certain mRNAs are incorporated into
P bodies only to be later released and utilized for protein translation.
(Choices A and B) The 5' end of all mRNA is capped with a 7-methylguanosine residue
by a unique 5' to 5' linkage, which occurs in two stages. The first step is the
addition of guanine triphosphate to the 5' end of mRNA in a reaction catalyzed by
guanylyltransferase. Methylation of the guanosine cap is then catalyzed by guanine-
7-methyltransferase. Capping of the precursor RNA occurs in the nucleus as the RNA
is being transcribed. This methylated cap protects mRNA from degradation by
cellular exonucleases, and allows it to exit the nucleus.
(Choice C) mRNA is polyadenylated at the 3' end by the polyadenylate polymerase
complex, which recognizes a specific sequence (AAUAAA), cleaves the pre-mRNA
molecule a few residues downstream from this sequence, and then adds a stretch of
20 - 250 adenine residues called the poly A tail. The addition of the poly A tail
occurs before the mRNA exits the nucleus. In the cytosol, the poly A tail is
gradually shortened, eventually leading to mRNA degradation.
(Choices D and E) Since pre-mRNA contains both introns and exons, and only exons
code for proteins, introns must be excised before translation through a process
known as splicing. Splicing of pre-mRNA occurs within the nucleus and is
facilitated by the interaction of pre-mRNA with small ribonucleoprotein particles
called snRNPs (or "snurps" for short).
Educational objective:
When mRNA is first transcribed from DNA, it is in an unprocessed form called pre-
mRNA or heterogeneous nuclear mRNA (hnRNA). Several processing steps are required
before finalized mRNA molecules can leave the nucleus, including 5'-capping, poly A
tail addition, and intron splicing. Cytoplasmic P bodies play an important role in
mRNA translation regulation and mRNA degradation.

RNA molecules that carry out functions without first being translated into proteins
are referred to as noncoding RNA Some important forms of non-coding RNA include
small nuclear RNA (snRNA), ribosomal RNA (rRNA), and transfer RNA (tRNA). Small
nuclear RNA molecules are transcribed by RNA polymerase II or III and are typically
associated with specific proteins forming small nuclear ribonucleoproteins (snRNPs,
or "snurps"). A collection of snRNPs on pre-mRNA is referred to as a spliceosome
These spliceosomes remove intron sequences from pre-mRNA by cleaving the 5' end of
the intron and joining that end to the branch point. The 3' end is subsequently
cleaved with the free ends of the remaining exon mRNA and ligated with a
phosphodiester linkage. Anti-snRNP antibodies are present in mixed connective
tissue disease.
(Choice A) Peroxisomes are cytoplasmic organelles containing oxidative enzymes such
as catalase, D-amino acid oxidase, and uric acid oxidase. These organelles are
ubiquitous among eukaryotes and are most abundant in the liver and kidneys where
detoxification of ingested and environmental materials occurs. In the liver,
peroxisomes also play a role in the breakdown of fatty acids.
(Choice B) Degradation of proteins and polypeptides occurs mainly in proteasomes
and lysosomes. Proteasomes mainly degrade intracellular proteins, while lysosomes
degrade extracellular proteins.
(Choice D) Eukaryotic chromatin is composed of repeated subunits called
nucleosomes, which consist of histone protein cores around which double stranded
DNA is wrapped Nucleosomes are important for the compact packaging of dsDNA into
chromosomes with the aid of other packaging proteins.
(Choice E) Ribosomes are present in the cytoplasm and are required for the
translation of mRNA into protein The molecule rRNA, along with ribosomal protein,
comprises individual ribosomes Within the cell, rRNA engages mRNA and facilitates
the entry of tRNA during the formation of polypeptide chains.
Educational Objective:
Small nuclear ribonucleoprotein particles (snRNPs) are important components of the
spliceosome, a molecule which functions to remove introns from pre-mRNA during
processing within the nucleus.

DNA polymerases are the main enzymes responsible for DNA replication. In E. coli,
there are three major DNA polymerases: I, II, and III Other enzymes that are
essential for the replication of DNA include: primase, helicase, ligase, and
topoisomerase I and II. During DNA replication, new daughter strands are
synthesized in the 5' to 3' direction, using the parent strands as complimentary
templates. Synthesis of DNA in the 3' to 5' direction does not occur in this
process.
Before the process of replication begins, the parent DNA double helix is unwound
and separated by the enzyme helicase. Helicase binds DNA at the origin of
replication with the assistance of DnaA protein and acts at the replication fork to
separate DNA This separation and unwinding of the DNA produces positive
supercoiling that can lead to DNA fracture, if not relieved. Topoisomerases I and
II (II is also known as gyrase) relieve unwinding tension.
DNA polymerases can not begin synthesis of daughter strands without a free 3'-
hydroxyl group, which is provided by an RNA primer and synthesized by the enzyme
primase (DNA dependent RNA polymerase).
DNA replication then proceeds with the leading strand being formed continuously in
the 5' to 3' direction toward the replication fork, and the lagging strand being
formed discontinuously in the 5’ to 3' direction, away from the replication fork.
Replication of the lagging strand results in the formation of numerous short DNA
segments, called Okazaki fragments, and each separate DNA segment requires a new
RNA primer upon which to initiate synthesis. The fragments of the lagging strand
are ultimately bound together by ligase after numerous RNA primers have been
removed and replaced with DNA. The removal of RNA primers is accomplished by DNA
polymerase I, the only bacterial DNA polymerase with 5' to 3’ exonuclease activity.
This activity allows DNA polymerase I to function both as an excision-repair enzyme
and as the enzyme that removes RNA primers.
Educational Objective:
Bacterial DNA polymerase I has 5' to 3' exonuclease activity, which is used to
excise RNA primers. The gaps created after RNA excision are then replaced with DNA
in the 5' to 3' direction by DNA polymerase I.

Transfer RNA (tRNA) is one form of non-coding RNA composed of between 74-93
nucleotides Specific molecules of tRNA transfer certain amino acid residues to the
growing polypeptide chain during translation. The tRNA molecule functions by
recognizing the three base codon on the mRNA molecule being translated through its
anticodon region, which contains complementary bases. The secondary structure of
tRNA resembles a cloverleaf and contains the following regions:
• The acceptor stem is created through the base pairing of the 5'-terminal
nucleotides with the 3'-terminal nucleotides. The CCA tail hangs off the 3' end,
with the amino acid bound to the 3' terminal hydroxyl group. tRNA is "loaded" with
the appropriate amino acid through the process of aminoacylation, which is
catalyzed by aminoacyl tRNA synthetase. The acceptor stem helps to mediate correct
tRNA recognition by the proper aminoacyl tRNA synthetase.
• A 3‘ CCA tail is added to the 3' end of tRNA as a posttranscriptional
modification in eukaryotes and in most prokaryotes. In some prokaryotic tRNAs, the
tail region is directly transcribed from the genome. Several enzymes utilize this
tail to help recognize tRNA molecules, and its presence is necessary for protein
translation.
• The D arm contains numerous dihydrouracil residues, which are modified bases
that are often present in tRNA. The D arm (along with the acceptor stem and
anticodon arm) facilitates correct tRNA recognition by the proper aminoacyl tRNA
synthetase.
• The anticodon arm contains sequences which are complementary to the mRNA
codon and is read in the 3’ to 5' direction. During translation, the ribosome
complex selects the proper tRNA molecule based solely upon its anticodon sequence.
• The T arm contains the TH^C sequence that is necessary for binding of tRNA to
ribosomes. The WC sequence refers to the presence of thymidine, pseudouridine, and
cytidine residues in this arm of tRNA. tRNA is the only RNA species that contains
the nucleoside thymidine.
• A 5‘ terminal phosphate
(Choice A) A TATA box is an upstream promoter region associated with some genes in
eukaryotic organisms. TATA binding protein binds to this promoter during
transcription, unwinding the DNA and initiating separation of the strands.
(Choices C and D) After transcription, eukaryotic pre-mRNA undergoes
posttranscriptional modification. The maturation process of precursor mRNA includes
the addition of a poly A tail at the 3' end and a methylguanosine cap at the 5' end
as well as the removal of introns (non-coding RNA).
(Choices E and F) AUG and UAG are mRNA start and stop codons that initiate and
terminate translation, respectively
Educational objective:
tRNA is a small, noncoding form of RNA that contains unusual nucleosides such as
pseudouridine and thymidine. Remember that tRNA has a CCA sequence at its 3'-end
that is used as a recognition sequence by proteins, and that the 3' terminal
hydroxyl group of the CCA tail is used as the binding site for the amino acid.

Although rare in the United States and other industrialized nations, vitamin A
deficiency is relatively common in Asia, Africa, and South America. Malnourishment
and fat malabsorption (eg, cystic fibrosis, cholestatic liver disease) are the most
significant causes of vitamin A deficiency. Clinical manifestations of the
condition include night blindness, complete blindness, and xerophthalmia More
unusual findings include Bitot's spots (abnormal squamous cell proliferation and
keratinization of the conjunctiva), corneal perforation, keratomalacia, nonspecific
dermatologic abnormalities, and humoral and cell-mediated immune system inhibition
via damage done to phagocytes and T cell lymphocytes. Death can result if the
condition is untreated.
Research has demonstrated that vitamin A is of benefit in treating children
afflicted with measles. Specifically, vitamin A therapy reduces the time to
recovery from pneumonia and diarrhea as well as the length of hospital stay and
risk of death. It has been postulated that the vitamin A resolves a virally-induced
hyporetinemia, though this has not yet been confirmed The WHO recommends that
vitamin A be administered to all children with measles in areas with widespread
vitamin A deficiency or in areas with a measles mortality rate in excess of one
percent.
(Choices B, C, D, E, and F) Vitamins K, D, E. and B do not appear to be of direct
benefit in the treatment of measles infection.
Educational Objective:
Vitamin A can be of benefit in the treatment of measles infection.

Marked riboflavin deficiency is rare in the United States, but can be seen in
chronic alcoholics and the severely malnourished Clinical manifestations of marked
riboflavin deficiency include angular stomatitis, cheilitis, glossitis, seborrheic
dermatitis, eye changes (eg, keratitis, corneal neovascularization), and anemia The
diagnosis is established with performance of the erythrocyte glutathione reductase
assay or evaluation of the urinary riboflavin excretion.
Metabolic modifications of riboflavin occur most frequently in the cells of the
heart, liver, and kidney. Typically, riboflavin is first phosphorylated to become
the coenzyme flavin mononucleotide (FMN). It can then either be integrated into a
coenzyme-flavin complex or can be further phosphorylated into flavin adenine
dinucleotide (FAD). FMN and FAD are required cofactors for flavoproteins, which are
enzymes that participate in numerous reduction-oxidation reactions within the human
body In the course of these reactions, the FMN and FAD cofactors are transformed
into their reduced, energy-carrying states (FMNH, and FADH,) through the acceptance
of electrons.
The riboflavin-containing coenzymes are key constituents of the electron transport
chain: FMN is a component of complex I, while FAD is a component of complex II FAD
is an electron carrier in the tricarboxylic acid cycle (TCA) and serves as a
cofactor for succinate dehydrogenase, which is an enzyme that mediates the
conversion of succinate into fumarate.
(Choices A, C, D, and E) Isocitrate dehydrogenase, succinate thiokinase, malate
dehydrogenase, and fumarase are enzymes that participate in the tricarboxylic acid
cycle (TCA). These enzymes do not use FAD or FMN as cofactors.
(Choice F) Glucose-6-phosphate dehydrogenase (G6PD) is the rate-limiting enzyme in
the pentose phosphate pathway. This pathway supplies NADPH for glutathione
reduction in RBCs.
(Choice G) HMG-CoA reductase is the rate-limiting enzyme in the cholesterol
synthesis pathway. FMN and FAD are not used as cofactors.
Educational Objective:
Riboflavin (vitamin B2) is a precursor of the coenzymes FMN and FAD. FAD
participates in tricarboxylic acid cycle as a coenzyme of succinate dehydrogenase,
which converts succinate into fumarate.

Pellagra (which means "rough skin" in Italian vernacular) is a clinical syndrome


arising secondary to niacin deficiency that is characterized by the "three Ds":
dermatitis, diarrhea, and dementia. The dermatitis is usually bilateral and
symmetric on the sun-exposed areas of the body, consisting of roughened, thickened,
and scaly skin. The diarrhea arises as a result of columnar epithelium atrophy (and
occasionally ulceration) of the gastrointestinal tract. The dementia develops
secondary to neuronal degeneration in the brain and spinal cord, with lesions
similar in appearance to those associated with pernicious anemia.
Niacin (nicotinic acid, or vitamin B3) is an essential component of the coenzymes
nicotinamide adenine
dinucleotide (NAD) and nicotinamide adenine dinucleotide phosphate (NADP). As such,
niacin plays a pivotal role in the intermediary metabolism of cells, serving to
accept electrons or donate hydrogen ions. Specifically, NAD functions as a coenzyme
for dehydrogenases involved in the metabolism of fats, carbohydrates, and amino
acids, NADP, in contrast, is crucial in the hexose-monophosphate shunt of glucose
metabolism.
Niacin is either obtained through dietary consumption (eg, grains, legumes, seed
oils, fruits, vegetables, meats) or is synthesized endogenously from tryptophan.
Because the niacin in corn is in a bound, unabsorbable form, populations that
primarily subsist on corn are prone to developing pellagra. In developed countries,
pellagra is primarily observed in alcoholics and those suffering from chronic,
debilitating diseases (eg, HIV infection). Pellagra can also be seen less commonly
in those with carcinoid syndrome, those using isoniazid for prolonged periods, and
those with Hartnup disease.
Educational Objective:
Niacin (vitamin B3) can be synthesized endogenously from tryptophan. A deficiency
of this vitamin results pellagra, which is characterized by dermatitis, diarrhea,
and dementia

Lactose (galactosyl beta-1,4-glucose or milk sugar) is a disaccharide present in


milk. It is synthesized in the mammary gland by formation of a 1,4 glycosidic
linkage between glucose and galactose Lactose in the diet is catabolized into
glucose and galactose by an intestinal brush-border disaccharidase called lactase
(a type of beta-galactosidase more specifically known as lactase-phlorizin
hydrolase). Lactose intolerance is characterized by gastrointestinal upset upon
ingestion of foods containing lactose, such as dairy products, and is caused by
deficiency of lactase (Answer E). Primary lactose intolerance is a very common
disorder, particularly in people of African and Asian descent. In contrast to most
other races, subjects of Northern European descent maintain lactase activity
throughout their life.
Secondary lactase deficiency occurs in association with a number of small
intestinal mucosal diseases such as celiac sprue and viral gastroenteritis. The
underlying pathophysiology of this disorder is due to the fact that lactase is
concentrated within epithelial cells in the microvilli of the small intestine (the
brush border). When these cells are damaged in gastroenteritis, the damaged cells
slough off and are replaced by immature cells that have low concentrations of
lactase.
The other answer choice options are important in the metabolism of galactose to
either glucose or lactose. Galactose is first phosphorylated to galactose-1-
phosphate by the enzyme galactokinase (Choice B) Next, galactose-1-phosphate uridyl
transferase (GALT) catalyzes the conversion of UDP-glucose and galactose-1-
phosphate to UDP-galactose and glucose-1-phosphate (Choice C). UDP-galactose is
then epimerized to UDP-glucose by UDP-galactose-4 epimerase, after which it can
participate in the appropriate glucose-related metabolic pathways. Alternatively,
UDP-galactose can be converted to galactosyl beta-1,4-glucose (lactose) by lactose
synthase within the mammary glands as part of the formation of milk (Choice D)
Galactosemia is an illness that is distinct from lactose intolerance, and it is
characterized by symptoms that start soon after the initiation of breast-feeding
Galactosemia can be caused by a deficiency of GALT (Type 1), galactokinase (Type
2), or UDP-glucose 4-epimerase (Type 3). Excess galactose in patients with
galactosemia is converted to galactitol by aldose reductase (Choice A), and high
levels of galactitol are responsible for many of the symptoms associated with
galactosemia (especially cataract formation).
Educational objective:
Secondary lactase deficiency can occur after viral gastroenteritis or other
diseases that damage the intestinal epithelium. This disease causes abdominal
distention, flatulence, and diarrhea after lactose ingestion.

The metabolism of fructose involves the enzymes fructokinase, aldolase B and


triokinase. Fructokinase deficiency causes essential fructosuria, a rare autosomal
recessive asymptomatic disorder. Fructose from the diet is absorbed in the gut and
secreted in the urine unchanged due to defective metabolism Fructose, similar to
glucose and galactose, is a reducing sugar and can be detected by clinitest
tablets, which test nonspecifically for the presence of reducing sugar A urine
dipstick, however, utilizes glucose oxidase for determination of the presence of
urine glucose and will not test positive in the presence of fructose or galactose.
In contrast to essential fructosuria, which is an asymptomatic and benign disorder,
the enzymatic deficiencies described in the other choices are associated with
symptoms as described below.
(Choice A) Galactosemia is an autosomal recessive disorder characterized by
neonatal jaundice, bleeding diathesis, feeding intolerance, hypotension and death
if untreated. This clinical syndrome is caused by absent function of galactose-1-
phosphate uridyl transferase. Treatment is by elimination of all milk products from
the diet and feeding with soy-based infant formula.
(Choice B) Metabolism of fructose-1-phosphate by aldolase B generates dihydroxy
acetone phosphate (DFIAP) and glyceraldehyde. Aldolase B deficiency is a life-
threatening disorder that can be treated by eliminating dietary fructose. Patients
become symptomatic only after ingesting fructose-containing foods Infants commonly
present with failure to thrive, hepatomegaly, and cirrhosis.
(Choice D) Acid alpha glucosidase (or acid maltase) deficiency causes glycogen
storage disease II, Pompe disease. This disease does not present with hypoglycemia
but with hepatomegaly, cardiomegaly and an increased risk for cirrhosis.
(Choice E) Lactase is a mucosal enzyme responsible for the digestion of lactose.
Acquired lactase deficiency is the most common cause of selective carbohydrate
malabsorption. Patients with lactase deficiency typically complain of
gastrointestinal symptoms including gaseous distention and diarrhea after the
ingestion of dairy products.
Educational Objective:
Unlike hereditary fructose intolerance and classic galactosemia, essential
fructosuria is a benign disorder resulting from a defect or deficiency in the
enzyme fructokinase.

The metabolism of fructose involves the enzymes fructokinase, aldolase B and


triokinase. Fructokinase deficiency causes essential fructosuria, a rare autosomal
recessive asymptomatic disorder. Fructose from the diet is absorbed in the gut and
secreted in the urine unchanged due to defective metabolism Fructose, similar to
glucose and galactose, is a reducing sugar and can be detected by clinitest
tablets, which test nonspecifically for the presence of reducing sugar A urine
dipstick, however, utilizes glucose oxidase for determination of the presence of
urine glucose and will not test positive in the presence of fructose or galactose.
In contrast to essential fructosuria, which is an asymptomatic and benign disorder,
the enzymatic deficiencies described in the other choices are associated with
symptoms as described below.
(Choice A) Galactosemia is an autosomal recessive disorder characterized by
neonatal jaundice, bleeding diathesis, feeding intolerance, hypotension and death
if untreated. This clinical syndrome is caused by absent function of galactose-1-
phosphate uridyl transferase. Treatment is by elimination of all milk products from
the diet and feeding with soy-based infant formula.
(Choice B) Metabolism of fructose-1-phosphate by aldolase B generates dihydroxy
acetone phosphate (DFIAP) and glyceraldehyde. Aldolase B deficiency is a life-
threatening disorder that can be treated by eliminating dietary fructose. Patients
become symptomatic only after ingesting fructose-containing foods Infants commonly
present with failure to thrive, hepatomegaly, and cirrhosis.
(Choice D) Acid alpha glucosidase (or acid maltase) deficiency causes glycogen
storage disease II, Pompe disease. This disease does not present with hypoglycemia
but with hepatomegaly, cardiomegaly and an increased risk for cirrhosis.
(Choice E) Lactase is a mucosal enzyme responsible for the digestion of lactose.
Acquired lactase deficiency is the most common cause of selective carbohydrate
malabsorption. Patients with lactase deficiency typically complain of
gastrointestinal symptoms including gaseous distention and diarrhea after the
ingestion of dairy products.
Educational Objective:
Unlike hereditary fructose intolerance and classic galactosemia, essential
fructosuria is a benign disorder resulting from a defect or deficiency in the
enzyme fructokinase.

The lac operon is the sequence of the E. coli genome which is required for the
metabolism of lactose. The lac operon consists of a regulatory gene (/), promoter
region (p), operator region (o), and three structural genes (z, y, and a). The z
gene codes for 3-galactosidase (3-gal), which is primarily responsible for the
hydrolysis of lactose to glucose and galactose. The y gene codes for permease, a
transmembrane enzyme that increases the permeability of the cell to lactose. The a
gene encodes a 3-galactoside transacetylase, which transfers acetyl groups to 3-
galactosides and is unnecessary for lactose metabolism by E. coli.
In prokaryotes, one mRNA transcript contains the sequences for many proteins, and a
single mRNA molecule can be translated into multiple proteins or polypeptides. For
instance, all three proteins of the lac operon (3-galactosidase, permease, and
transacetylase) are synthesized from a single mRNA molecule containing the z, y and
a gene sequences, respectively. Transcription and translation of the genes of the
lac operon is typically synchronous. Remember that a single mRNA molecule which
codes for more than one protein is referred to as a polycistronic mRNA, and while
most prokaryotic mRNA molecules are polycistronic, eukaryotic mRNA is rarely
polycistronic.
(Choices A - D) The lac operon, which codes for all three aforementioned proteins,
is regulated by a single operator, a promoter, and a single group of regulatory
elements: an inducer, repressor, and catabolite activator protein. Modulation of
the transcription of this operon through binding of the operator and action of the
repressor or other regulatory elements will change the transcription of all
three /ac-operon structural genes (z, y, and a). On the other hand, there are no
operators, repressors, or inducers that can desynchronize the transcription of lac-
operon structural genes.
Educational Objective:
Bacterial mRNA can be polycistronic, meaning that one mRNA codes for several
proteins. An example of polycistronic mRNA is the bacterial lac operon, which codes
for the proteins necessary for lactose metabolism by E. coir, the transcription and
translation of these bacterial proteins is regulated by a single oromoter ODerator
and set of reaulatorv elements

Digestion and absorption of nutrients primarily occurs in the small intestine.


Small intestinal epithelial cells produce several enzymes responsible for nutrient
absorption. Proteins in ingested food exist primarily as polypeptides and require
hydrolysis to dipeptides, tripeptides and amino acids for absorption. Hydrolysis of
these polypeptides is accomplished by proteolytic enzymes such as pepsin and
trypsin These enzymes are secreted as the inactive proenzymes pepsinogen and
trypsinogen from the stomach and pancreas, respectively Trypsin, in turn, activates
other proteolytic enzymes including chymotrypsin, carboxypeptidase and elastase.
Activation of trypsinogen to trypsin is achieved by enteropeptidase (or
enterokinase), an enzyme produced in the duodenum Enteropeptidase deficiency
results in defective conversion of the proenzyme trypsinogen to the active enzyme
trypsin. Typical clinical manifestations of enteropeptidase deficiency are
diarrhea, growth retardation and hypoproteinemia.
(Choice A) Lipase secreted from the exocrine pancreas is the most important enzyme
for the digestion of triglycerides. Chronic pancreatitis is a painful condition
that causes lipase deficiency. This leads to poor fat absorption and steatorrhea
(foul smelling bulky stools containing undigested fat).
(Choice B) Activation of pepsinogen to pepsin requires an acidic pH and a small
amount of preexisting pepsin. Pepsin initiates the digestive process of proteins in
the stomach that is completed by trypsin and other enzymes in the proximal small
intestine.
(Choices C and E) Hydrolysis of complex carbohydrates to oligo-, di- and
monosaccharides is carried out by pancreatic amylase. Lactase produced in the
intestinal brush border is responsible for the conversion of lactose to glucose and
galactose. Deficiency of lactase is common and causes lactose intolerance.
(Choice F) Secretin is a peptide hormone secreted by the S-cells of the duodenum in
response to low duodenal pH. Secretin stimulates the secretion of bicarbonate from
the pancreas and gall bladder and reduces acid secretion in the stomach by reducing
the production of gastrin. Neutralizing the acidic pH of food entering the duodenum
from the stomach is necessary for the proper function of pancreatic enzymes
(amylase, lipase).
Educational Objective:
Trypsinogen is activated to trypsin by duodenal enteropeptidase. Trypsin is
essential for protein digestion and absorption in two ways. It degrades complex
peptides to dipeptides and amino acids, and it activates other proteases such as
carboxypeptidase, elastase and chymotrypsin.

Amatoxins are found in a variety of poisonous mushrooms (eg, Amanita phalloides.


known as death cap) and are responsible for the majority of mushroom poisoning
fatalities worldwide. Ingestion of 1 or more amatoxin-containing mushrooms is a
life-threatening emergency. After absorption by the gastrointestinal tract,
amatoxins are transported to the liver via the portal circulation where active
transport by organic anion transporting polypeptide (OATP) and sodium taurocholate
co-transporter (NTCP) concentrates the toxin within the liver cells. There,
amatoxins bind to DNA-dependent RNA polymerase type II and halt mRNA synthesis,
ultimately resulting in apoptosis. Other organ systems with rapid cellular turnover
can also be affected in amatoxin poisoning, including the gastrointestinal tract
and proximal convoluted renal tubules.
Symptoms typically start 6-24 hours after ingestion and include abdominal pain,
vomiting, and severe, choleralike diarrhea that may contain blood and mucus. Severe
poisoning can lead to acute hepatic and renal failure. Urine testing for a-amanitin
can confirm suspected amatoxin poisoning
(Choice A) Acyclovir and related drugs (eg, famciclovir and valacyclovir) are
inhibitors of viral DNA polymerase.
(Choice C) Ricin (from the castor oil plant Ricinus communis) is a potent toxin
that inhibits protein synthesis by cleaving the rRNA component of the eukaryotic
60S subunit.
(Choice D) The only function of RNA polymerase I is to transcribe the majority of
the eukaryotic ribosomal RNA components. RNA polymerase I is insensitive to
amatoxins.
(Choice E) Eukaryotic RNA polymerase III transcribes transfer RNA, 5S ribosomal
RNA. and other small RNA molecules. It is only weakly affected by amatoxins.
Educational objective:
Amatoxins are found in a variety of poisonous mushrooms (eg, Amanita phalloides,
known as death cap) and are potent inhibitors of RNA polymerase II (halting mRNA
synthesis).

Leptin is a protein hormone that plays an important role in regulating appetite and
metabolism. It is produced primarily in adipocytes, and large fat cells produce
more leptin than small ones. Serum leptin concentrations are highly correlated with
body fat content.
Leptin decreases food intake in the following important ways:
1. Leptin decreases the production of neuropeptide Y, a potent appetite
stimulant, in the arcuate nucleus of the hypothalamus
2. Leptin stimulates the production of proopiomelanocortin (POMC) in the arcuate
nucleus. Alpha-melanocyte-stimulating hormone (alpha-MSH) is produced by cleavage
of POMC and inhibits food intake.
The knockout mouse described is homozygous for a mutation in the gene encoding the
leptin receptor (db/db), resulting in ineffective leptin signaling As a result,
these mice become hyperphagic and profoundly obese As leptin production is normal
in these mice, leptin levels are elevated due to the increased lipocyte mass. In
contrast, mice that are homozygous for a mutation resulting in impaired leptin
production (ob/ob) also become hyperphagic and profoundly obese, but their leptin
levels are low (Choice A)
Human obesity resulting from mutations in the leptin receptor and the leptin gene
has been described. However, most obese individuals do not have either of these
mutations. Instead, it is thought that the sustained elevation in leptin levels
from the enlarged fat stores results in leptin desensitization. Thus, obese
individuals become resistant to the effects of leptin in a manner similar to the
development of insulin resistance in type 2 diabetes.
(Choice D) Low leptin and low BMI correlate with low adipocyte stores and may be
seen after prolonged starvation.
(Choice E) With intact receptor signaling, elevated leptin levels (ie, from
exogenous leptin administration) would result in weight loss.
Educational objective:
Leptin is a protein hormone produced by adipocytes in proportion to the quantity of
fat stored. Leptin acts on the arcuate nucleus of the hypothalamus to inhibit
production of neuropeptide Y (decreasing appetite) and stimulate production of
alpha-MSH (increasing satiety). Mutations in the leptin gene or receptor result in
hyperphagia and profound obesity.

This patient has hepatic encephalopathy, likely due to recent gastrointestinal


bleeding and a corresponding increase in ammonia and nitrogen absorption by the
gut. The pathogenesis of hepatic encephalopathy is related to increased circulatory
levels of ammonia and other neurotoxins due to failure of the liver to metabolize
waste products. When ammonia levels rise acutely, astrocyte and neuron function are
affected.
Within the brain, astrocytes and neurons interact to regulate the metabolism of
glutamate, glutamine, and ammonia in a process known as the glutamate-glutamine
cycle Glutamate released by neurons during neurotransmission is taken up by
astrocytes and converted to glutamine, a non-neuroactive compound Glutamine is then
released by astrocytes and taken up by neurons, where it is either converted back
to glutamate for use as a neurotransmitter or transaminated into a-ketoglutarate
for use in the Krebs cycle.
Ammonia toxicity results in part from depletion of glutamate and alpha-
ketoglutarate in the brain during the process of ammonia detoxification.
Hyperammonemia increases the conversion of glutamate into glutamine by glutamine
synthetase within astrocytes (Choice C) The resulting increase in glutamine leads
to hyperosmolarity and mitochondrial dysfunction, causing astrocytic swelling and
impairment. Increased glutamine formation also decreases total brain glutamate
stores, impairing excitatory neurotransmission (mediated by NMDA, AMPA, and kainate
receptors) and neuronal energy production. In addition, ammonia can be detoxified
to glutamate via glutamate dehydrogenase, depleting a-ketoglutarate and further
impairing energy metabolism in the brain
(Choice B) Carnitine is responsible for transport of fatty acids into the
mitochondria for beta-oxidation. Impaired beta-oxidation of fatty acids to acetyl-
CoA can occur with excessive alcohol consumption.
(Choice D) Elevated lactate can be seen in patients with cirrhosis, particularly in
conditions that cause elevated lactate production (eg, sepsis, tissue
hypoperfusion). The liver metabolizes lactate, and the lack of functioning
hepatocytes in cirrhosis decreases hepatic lactate disposal.
(Choice E) Oxindole is a tryptophan derivative formed by bacteria in the gut and
normally cleared by the liver. It causes sedation, muscle weakness, hypotension,
and coma. Elevated levels of oxindole have been found in patients with hepatic
encephalopathy.
Educational objective:
Hyperammonemia in hepatic encephalopathy results in depletion of a-ketoglutarate,
causing inhibition of the Krebs cycle. Excess ammonia also depletes glutamate, an
excitatory neurotransmitter, and causes accumulation of glutamine, resulting in
astrocyte swelling and dysfunction.

DNA can be damaged in a number of ways (spontaneous mutations, chemical reactions,


or ultraviolet / ionizing radiation) through the following mechanisms:
1. Depurination of DNA and base excision repair (spontaneous or chemical)
2. Formation of thymine dimers (ultraviolet rays)
3. Breaks in DNA chains and oxidative damage (ionizing radiation)
4. Cross-linkage, intercalation, alkylation (chemical / pharmacologic agents)
The DNA repair mechanism utilized by cells for base alterations is called base
excision repair (not to be confused with nucleotide excision repair). Nitrates
consumed in the diet lead to the deamination of cytosine, adenine, and guanine to
form uracil, xanthine, and hypoxanthine. respectively These resulting bases are not
normally present in DNA and are recognized by specific glycosylases, which cleave
these altered DNA bases from the parent DNA molecule This leaves an empty sugar-
phosphate site called a basic site, or apurinic-apyrimidinic site (AP). An
endonuclease then cleaves the 5' end of the AP site before a lyase enzyme
subsequently completes the removal of the AP site from the DNA molecule by removing
the sugar phosphate group. DNA polymerase then fills the gap with the correct
sugar-phosphate-base, which is finally joined to the strand by ligase (Choice A).
Educational Objective:
Base excision repair is used to correct defects in single bases induced
spontaneously or by exogenous chemicals. In this process, glycosylases remove the
defective base, and the corresponding sugar-phosphate is cleaved and removed by
endonuclease, followed by the action of lyase. DNA polymerase then replaces the
missing nucleotides and ligase reconnects the DNA strand

The initiation of transcription in prokaryotes is controlled by two upstream DNA


sequences, known as promoter sequences, which promote transcription by RNA
polymerase. There are a number of accessory proteins that regulate the interaction
of RNA promoters at the transcription site. These proteins can work as activators
or repressors of gene transcription. "Operon" is the term used to describe a DNA
sequence that contains at least two regulatory sites, the promoter and the
regulator (operator), in addition to the gene coding for the structure of the
protein itself.
The lac operon is the genetic sequence in the E. coli genome that codes for
proteins required for the metabolism of lactose. The lac operon consists of a
regulatory gene (/), promoter region (p), operator region (o), and three structural
genes (z, y, and a). The z gene codes for [3-galactosidase ((3-gal), which is
primarily responsible for the hydrolysis of lactose to glucose and galactose. The y
gene codes for permease, a transmembrane enzyme that increases the permeability of
the cell to lactose. The i gene codes for the repressor protein of the lac operon,
which binds the DNA sequence at the operator to prevent binding of RNA polymerase
to the promoter. The p gene is the promoter region, the initial binding site for
RNA polymerase, to initiate transcription of the lac operon.
Culturing E. coli in lactose-containing media results in the binding of lactose to
the repressor protein. This binding causes a conformational change which prevents
the attachment of the repressor protein to the operator region, which, in turn,
increases transcription of the lac operon structural genes and. subsequently,
increases utilization of lactose for energy. Culturing E. coli in glucose-
containing media with or without lactose represses the expression of the lac
operon. Glucose decreases the activity of adenylyl cyclase and leads to a reduction
in intracellular cAMP. Depletion of cAMP by the presence of glucose decreases the
expression of the lac operon structural genes High cAMP levels activate a protein
called catabolite activator protein (CAP), which binds cAMP to form a cAMP-CAP
complex which then binds to a region upstream from the promoter region and acts as
positive regulator of the lac-operon The main mechanism of glucose-induced
inhibition of lac operon expression is depletion of cAMP (Choice E), not binding of
the repressor protein to the operator (Choice A)
(Choice B) Repressor proteins do not bind to promoter region; they bind the
operator region.
(Choices C and D) In contrast to lactose, which acts as an inducer, glucose does
not bind to the repressor protein or promoter.
Educational Objective:
Glucose induced decreased adenylate cyclase activity leads to low intracellular
concentrations of cAMP. Low cAMP levels, in turn, cause poor binding of catabolite
activator protein (CAP) to the CAP-DNA binding domain, leading to decreased
expression of the structural genes of the lac operon.

The lac operon consists of a regulatory gene (lac I), a promoter region (lac p), an
operator region (lac o), and three structural genes (lac Z, lac Y. and lac A). The
lac Z gene codes for P-galactosidase, which is responsible for the hydrolysis of
lactose to glucose and galactose. The lac Y gene codes for permease, which allows
lactose to enter the bacterium. The lac p region is the binding site for RNA
polymerase during the initiation of transcription The Lac I repressor protein is
the product of the lac I gene and is constitutively expressed Repressor proteins,
when bound to the operator region, prevent binding of RNA polymerase to the
promoter region, thus decreasing transcription of the lac Z, lac Y, and lac A
genes. Culture of E coll in lactose-containing media causes a conformational change
in the repressor protein, preventing its attachment to the operator region and
increasing transcription of the lac operon structural genes.
Culturing E coli in media containing glucose results in reduced expression of the
lac operon, even when the media contains lactose as well. This occurs because the
lac operon is positively regulated by the binding of catabolite activator protein
(CAP) to a site slightly upstream from the promoter region. This only occurs when
cAMP concentrations are high. Since glucose decreases the activity of adenylyl
cyclase (reducing intracellular cAMP), the lac operon is repressed in high-glucose
conditions. In summary, the lac operon is regulated by 2 distinct mechanisms:
1. Negatively by binding of the repressor protein to the operator locus
2. Positively by cAMP-CAP binding upstream from the promoter region
Mutations impairing the binding of the repressor protein to its binding site at the
operator region will prevent repression of the genes of the lac operon in the
absence of lactose. This results in increased transcription of the genes of the lac
operon in lactose-deficient media, although the presence of glucose will prevent
maximal transcriptional activity.
(Choices A and D) Mutations that impair the binding of cAMP-CAP to its regulatory
site upstream from the promoter will decrease transcription of the lac operon, as
cAMP-CAP is a positive regulator.
(Choices C and E) Mutations impairing the binding of RNA polymerase to the promotor
region will also reduce transcription of the lac operon.
Educational objective:
The lac operon is regulated by two distinct mechanisms: negatively by binding of
the repressor protein to the operator locus and positively by cAMP-CAP binding
upstream from the promoter region. Constitutive expression of the structural genes
of the lac operon occurs with mutations that impair the binding of the repressor
protein (Lac I) to its regulatory sequence in the operator region

Culturing E coli in media containing glucose results in reduced expression of the


lac operon, even when the media contains lactose as well. This occurs because the
lac operon is positively regulated by the binding of catabolite activator protein
(CAP) to a site slightly upstream from the promoter region. This only occurs when
cAMP concentrations are high. Since glucose decreases the activity of adenylyl
cyclase (reducing intracellular cAMP), the lac operon is repressed in high-glucose
conditions. In summary, the lac operon is regulated by 2 distinct mechanisms:
1. Negatively by binding of the repressor protein to the operator locus
2. Positively by cAMP-CAP binding upstream from the promoter region
Mutations impairing the binding of the repressor protein to its binding site at the
operator region will prevent repression of the genes of the lac operon in the
absence of lactose. This results in increased transcription of the genes of the lac
operon in lactose-deficient media, although the presence of glucose will prevent
maximal transcriptional activity.
(Choices A and D) Mutations that impair the binding of cAMP-CAP to its regulatory
site upstream from the promoter will decrease transcription of the lac operon, as
cAMP-CAP is a positive regulator.
(Choices C and E) Mutations impairing the binding of RNA polymerase to the promotor
region will also reduce transcription of the lac operon.
Educational objective:
The lac operon is regulated by two distinct mechanisms: negatively by binding of
the repressor protein to the operator locus and positively by cAMP-CAP binding
upstream from the promoter region. Constitutive expression of the structural genes
of the lac operon occurs with mutations that impair the binding of the repressor
protein (Lac I) to its regulatory sequence in the operator region

Growth factors can stimulate cell proliferation by altering the expression of


certain genes. This requires the use of signal transduction systems that can
transfer the signal to the nucleus. Examples of such systems include:
1. Ras-MAP kinase pathway
2. PI3K/Akt/mTOR pathway
3. Inositol phospholipid pathway
4. cAMP pathway
5. JAK/STAT pathway
The Ras-MAP kinase pathway starts with a growth factor ligand binding to the
receptor tyrosine kinase, causing auto-phosphorylation of the receptor.
Phosphotyrosine produced in this reaction interacts with a number of proteins (such
as SH2-domain proteins and SOS protein), leading to Ras activation Ras is a G-
protein that exists in active and inactive forms. Inactive Ras contains GDP, while
the active form is bound to GTP. Activated Ras begins a phosphorylation cascade
starting with activation of Raf kinase. This cascade results in the activation of
MAP (mitogen-activated protein) kinase which enters the nucleus to influence gene
transcription.
The Ras protein exists in a balance between its active and inactive forms. Inactive
(GDP-containing) Ras is activated by a signal originating from the receptor
tyrosine kinase. Active (GTP-containing) Ras is inactivated by GAP (GTPase-
activating protein), which induces the hydrolysis of GTP into GDP. Mutation of Ras
can lead to an inability to split GTP; the resultant permanently activated Ras
stimulates cell proliferation and can lead to cancer
(Choice A) ATP is not a messenger in signal transduction pathways.
(Choice B) cAMP activates cAMP-dependent protein kinase (protein kinase A) in the
cAMP transduction pathway.
(Choices C and E) The inositol-lipid pathway uses IP3 as a messenger. IP. causes
Ca:+ release from the endoplasmic reticulum.
Educational objective:
The MAP-kinase signal transduction pathway includes Ras protein, a G-protein that
exists in inactive (GDP-containing) and active (GTP-containing) forms. Mutated
(permanently activated) Ras is associated with the development of malignant tumors.

There are 64 codons in the genetic code, the majority of which code for amino
acids. Because there are only 20 amino acids, most amino acids have more than one
codon. For example. GUU, GUC, GUA and GUG all code for valine. In addition, there
are codons that call for the initiation and termination of protein synthesis. AUG,
which codes for methionine, is the universal start codon. UAA, UAG and UGA are stop
codons. The stop codons do not code for amino acids. Instead, when the ribosome
encounters a stop codon, releasing factors bind to the ribosome and stimulate
release of the formed polypeptide chain and dissolution of the ribosome-mRNA
complex.
(Choice A) Transcription produces a pre-mRNA molecule containing both introns and
exons. Posttranscriptional processing removes the introns. Splicing is accomplished
via small nuclear ribonucleoproteins (snRNPs).
(Choice B) Uncharged tRNA (without an amino acid) would not interact with mRNA and
ribosomes during protein synthesis.
(Choice C) Charged tRNA delivers amino acids to the protein synthesis complex. The
anticodon on a tRNA molecule recognizes the corresponding codon on mRNA, assuring
proper amino acid sequencing.
(Choices E and F) Initiation of gene transcription is governed by transcription
factor binding to the gene’s regulatory region. Transcription factor II D is a
transcription factor that binds to the TATA promoter region located approximately
25 bp upstream from the gene's coding region. Elongation factors facilitate tRNA
binding and the translocation steps of protein synthesis.
Educational Objective:
Releasing factors recognize the stop codons (UAA, UAG and UGA) to terminate protein
synthesis. They facilitate release of the polypeptide chain from the ribosome and
dissolution of the ribosome-mRNA complex.

Lead is a soft, heavy metal used in the manufacture of a wide range of items,
including batteries alloys, and ammunition. The most significant means of
occupational exposure is inhalation. Consumers can also be exposed to lead through
contact with dust in lead-painted homes (old homes), lead-contaminated soil, water
that traverses lead plumbing, lead-glazed pottery, and homemade alcoholic beverages
(moonshine). The most significant means of exposure for the consumer is
gastrointestinal absorption.
Lead overdose causes toxicity through four distinct mechanisms. Perhaps most
importantly, lead has a strong affinity for sulfhydryl groups. This affinity
results in the inhibition of enzymes that incorporate iron into the heme molecule
(delta-aminolevulinic acid dehydratase and ferrochetolase). Delta-aminolevulinic
acid is formed when succinyl-CoA and glycine combine in the presence of the
cofactor pyridoxal phosphate. The blood and urinary levels of this enzyme are
increased in individuals with lead poisoning.
(Choice A) Thiamine pyrophosphate is an essential vitamin B1-derived cofactor that
plays a role in multiple pathways of energy metabolism (e g., pyruvate and a-
ketoglutarate dehydrogenase catalyzed reactions).
(Choice C) Lipoic acid, or lipoate, is an essential cofactor in numerous facets of
aerobic metabolism (eg, transfer of acyl and methylamine groups in 2-oxoacid
dehydrogenase and glycine cleavage complexes, respectively). It is also an
antioxidant capable of scavenging reactive oxygen species and reducing metabolites.
(Choice D) Retinoic acid binds to receptor proteins, and the subsequent retinoic
acid-protein complex interacts with and affects expression of numerous genes that
affect growth and differentiation.
(Choice E) Biotin is the cofactor necessary for many reactions involving
carboxylation enzymes (e g., acetyl-CoA carboxylase and pyruvate carboxylase).
Educational Objective:
Pyridoxal phosphate is a necessary cofactor in the synthesis of delta-
aminolevulinic acid (which is elevated in cases of lead poisoning).

Lynch syndrome (hereditary nonpolyposis colon cancer) is an autosomal dominant


disease caused by defective DNA mismatch repair DNA replication occurs with a high
degree of fidelity because mismatched nucleotides are repaired through the
proofreading activity of DNA polymerases delta and epsilon. However, this
proofreading functionality is not infallible; base substitutions and small
insertion-deletion mismatches occur due to errors in base pairing every 106 bases
on average. It is the function of the DNA mismatch repair system to fix these
errors shortly after the daughter strands are synthesized. The mismatch repair
system involves several genes, including MSH2 and MLH1, which code for components
of the human MutS and MutL homologs. Mutations in these 2 genes account for around
90% of cases of Lynch syndrome.
Mismatch repair begins with MutS homolog detecting a mismatch on the newly created
daughter strand, which is distinguished from the parent strand by occasional nicks
in the phosphodiester bonds. MutL homolog is then recruited, and the resulting
complex slides along the DNA molecule until 1 of the daughter strand nicks is
encountered. At this point, exonuclease 1 is loaded onto and activated by the
repair complex. The daughter strand is then degraded backward past the initial
mismatch point, leaving a variable gap of single-stranded DNA that is stabilized by
ssDNA-binding protein. The complex then dissociates while DNA polymerase delta
loads at the 3' end of the discontinuity and begins synthesizing a new daughter
strand segment. Finally, DNA ligase I seals the remaining nick to complete the
repair process.
(Choice B) Exposure to ultraviolet light can cause pyrimidine (usually thymine)
dimers to form due to covalent joining of adjacent pyrimidines. Pyrimidine dimers
interfere with DNA replication and are removed by nucleotide excision repair.
(Choices C and D) Several types of insults can alter the DNA bases. For example,
nitrous acid can deaminate C, A, and G. There are also spontaneous changes, such as
deamination of C to U and the constant low-level loss of purines via thermal
disruption Glycosylases are enzymes that detect and remove abnormal bases from DNA.
creating an empty sugar-phosphate residue that is subsequently removed and replaced
by the correct nucleotide (base excision repair).
(Choice E) Exposure to ionizing radiation causes double-stranded DNA breaks that
are repaired by end-joining repair mechanisms. Non-homologous end joining, the main
mechanism in primates, is more prone to cause mutations than homologous
recombination.
Educational objective:
Lynch syndrome is an autosomal dominant disease caused by abnormal nucleotide
mismatch repair. The mismatch repair system involves several genes, including MSH2
and MLH1, which code for components of the human MutS and MutL homologs. Mutations
in these 2 genes account for around 90% of cases of Lynch syndrome.

The chronic myeloproliferative disorders are a group of bone marrow diseases


characterized by the overproduction of myeloid cells. Primary myelofibrosis is
caused by atypical megakaryocytic hyperplasia The clonally expanded megakaryocytes
activate fibroblast proliferation, resulting in progressive replacement of the
marrow space by extensive collagen deposition. In the early stages, there is marrow
hypercellularity with minimal fibrosis As the disease progress, pancytopenia can
result Hepatomegaly and massive splenomegaly occur in myelofibrosis because the
loss of bone marrow hematopoiesis is compensated for by extramedullary
hematopoiesis The peripheral smear characteristically shows teardropshaped red
blood cells (dacrocytes) and nucleated red blood cells.
With the exception of chronic myelogenous leukemia, the chronic myeloproliferative
disorders (especially polycythemia vera) frequently harbor a mutation in the
cytoplasmic tyrosine kinase, Janus kinase 2 (JAK2). This mutation (V617F)
substitutes a bulky phenylalanine for a conserved valine at position 617, resulting
in constitutive tyrosine phosphorylation activity, and consequently, cytokine-
independent activation of the JAK-STAT pathway. A JAK2 inhibitor (ruxolitinib) has
been approved for the treatment of primary myelofibrosis.
(Choice A) Acute lymphocytic leukemia predominantly affects children Clinical
manifestations are nonspecific and include fever, fatigue, pallor, petechiae, and
bleeding. Leukemic spread can cause lymphadenopathy, hepatosplenomegaly, and bone
pain
(Choice B) In acute promyelocytic leukemia, the translocation t(15;17) leads to the
formation of a fusion gene between the promyelocytic leukemia (PML) and the
retinoic acid receptor alpha (RARa) genes This abnormal PML/RARa fusion protein
blocks the differentiation of myeloid precursors.
(Choice C) Chronic lymphocytic leukemia is a lymphoproliferative disorder involving
B-lymphocytes. The most significant laboratory finding is marked lymphocytosis,
with “smudge cells" seen on peripheral blood smear. The majority of cases exhibit
increased expression of the proto-oncogene BCl-2, similar to follicular lymphomas
(Choice D) Several high-grade non-Hodgkin lymphomas are associated with cytogenetic
abnormalities. The t (8; 14) translocation is the most common in Burkitt lymphoma
and involves the c-myc oncogene Burkitt lymphoma is associated with Epstein-Barr
virus infection and classically has a "starry sky" histologic appearance

Das könnte Ihnen auch gefallen